You are on page 1of 37

Republic of the Philippines 12 kg to lbs.

Department of Education 2.2 𝑙𝑏𝑠


Region III – Central Luzon 1 1 𝑘𝑔 = 𝟐𝟔. 𝟑 𝒍𝒃𝒔
SCHOOLS DIVISION OF CITY OF BALANGA BATAAN 2
NATIONAL HIGH SCHOOL – SENIOR HIGH SCHOOL
City of Balanga, Bataan 1. 10 km to 𝑘 nm

� 1 1 𝑥 10 𝑚 𝑥
3 1 𝑛𝑚
= 𝟏𝟎 𝒙 𝟏𝟎𝟏𝟐 𝒏𝒎
ACTIVITY SHEETS IN GENERAL PHYSICS 1 0 1 𝑘𝑚 1
𝑥𝑘 𝑥
� 1
� 0−
9
𝑥
Quarter 1 - Week 1, Day 1- 4 (Solve measurement problems 𝑚 a measurement that is
Scientific notation is also a way of writing
involving conversion of units, expression of measurements in too large or too small. In order to express the units in scientific
scientific notation, differentiate accuracy from precision, notation, a non-zero digit should be placed before the decimal
differentiate random errors from systematic errors, estimate
point and all of the remaining significant figures will be decimals.
errors from multiple measurements of a physical quantity
using variance, differentiate vector and scalar quantities, 15 kg to milligram
perform addition of vectors and rewrite a vector in component
form) 1 𝑥 1 𝑚𝑔
Units are used to determine the number of measured qualities 1 10 𝑔 𝑥
3
= 15𝑥106𝑚𝑔 = 𝟏. 𝟓𝒙𝟏𝟎𝟕𝒎𝒈
5 1 𝑘𝑔 1
based on a set standard. There are several systems of units but the
𝑘 𝑥
most common used are the SI units and British units. Units can be Accuracy and Precision 1
converted within the same system units or across other. 𝑔
𝑥 0−
3
𝑔
system units. Accuracy refers to how close a measurement is to the true or
accepted value while Precision refers to how close measurements
Here is a list of few common units and its conversion to of the same item are close to each other. Here are some examples
other system units that shows the difference between accuracy and precision..

If the measurement acquired is too small or too large from its


Errors and uncertainties
base unit, prefixes are used. Here are some prefixes used.
Systematic errors are errors occurred due to misuse of the
measuring equipment. These errors may re occurred if the
equipment isn’t calibrated well.

Random errors are errors occurred unavoidably. These errors


may be due to the defect of the measuring tools when it was
manufactured or due to some random factors.

There are two ways in estimating errors from a multiple


measurement. First one is the conservative error estimate where
the maximum possible error is assumed.

Assumed for the following example that two measured quantities


are 𝑥 ± ∆𝑥 𝑎𝑛𝑑 𝑦 ± ∆𝑦 where ∆𝑥 𝑎𝑛𝑑 ∆𝑦 are uncertainties and ∆𝑧
is the total uncertainty.

Conservative error estimate for addition and subtraction:


Example:
If 𝑧 = 𝑥 + 𝑦 𝑜𝑟 𝑧 = 𝑥 − 𝑦 𝑡ℎ𝑒𝑛 ∆𝒛 = ∆𝒙 + ∆𝒚
1 𝑘𝑔 = 1 𝑘𝑖𝑙𝑜𝑔𝑟𝑎𝑚 = 1 𝑥 103 𝑔 = 1 000 𝑔
Conservative error estimate for multiplication and division:
1 𝑚𝑚 = 1 𝑚𝑖𝑙𝑖𝑚𝑒𝑡𝑒𝑟 = 1 𝑥 10−3 𝑚 = 0.001 𝑚 ∆𝒙 ∆𝒚
If 𝑧 = 𝑥𝑦 𝑜𝑟 𝑧 = 𝑥/𝑦 𝑡ℎ𝑒𝑛 ∆𝒛 = 𝒛 ( + )
Conversion of units 𝒙 𝒚

Both tables above can be used in converting units. The when the calculated or derived quantity is calculated based on a
following measurements are converted as an example. large number of other quantities a less conservative error estimate
is warranted:
1|Page
Nonconservative error estimate for addition and A paraglider started his flight heading south and took 5 km
subtraction: distance. He then turned west (assumed that the glider didn’t take
any distance while turning) and took another 5 km distance before
If 𝑧 = 𝑥 + 𝑦 𝑜𝑟 𝑧 = 𝑥 − 𝑦 𝑡ℎ𝑒𝑛 ∆𝒛 = √∆𝒙𝟐 + ∆𝒚𝟐
landing. Find its displacement (shortest distance between starting
Nonconservative error estimate for multiplication and point and end point).
division:

∆𝒚 𝟐
If 𝑧 = 𝑥𝑦 𝑜𝑟 𝑧 = 𝑥/𝑦 𝑡ℎ𝑒𝑛 ∆𝒛 = 𝒛√( ∆𝒙 𝟐 +( )
𝒙) 𝒚

Here are some examples for both conservative and nonconservative


error estimate:

Let 𝑥 = 8 ± 0.1𝑚 𝑎𝑛𝑑 𝑦 = 4 ± 0.2𝑚

Using conservative error estimate: x-component y-component


Vector Angle (ɵ) 𝒗 ∙ 𝐜𝐨𝐬 (𝜽) 𝒗 ∙ 𝒔𝒊𝒏 (𝜽)
1. 𝑧 = 𝑥 + 𝑦 ∴ ∆𝑧 = ∆𝑥 + ∆𝑦 = 0.1𝑚 + 0.2𝑚 = 0.3𝑚 5 km 270 0 km -5 km
𝒛 = 𝟏𝟐 ± 𝟎. 𝟑 𝒎 5 km 180 -5 km 0
2. 𝑧 = 𝑥 − 𝑦 ∴ ∆𝑧 = ∆𝑥 + ∆𝑦 = 0.1𝑚 + 0.2𝑚 = 0.3𝑚 Total: -5 km -5 km
𝒛=𝟒±𝟑𝒎
∆𝑥 ∆𝑦 0.1 0.2
3. 𝑧 = 𝑥𝑦 ∴ ∆𝑧 = 𝑧 ( + ) = 32 ( + ) = 2𝑚2
𝑥 𝑦 8 4 This indicates that the displacement of the paraglider is 𝑹 =
𝒛 = 𝟑𝟐 ± 𝟐 𝒎𝟐 −𝟓𝒌𝒎 𝒊̂ − 𝟓𝒌𝒎 𝒋̂.̂ This answer already shows the magnitude and
∆𝑥 ∆𝑦 0.1 0.2
4. 𝑧 = 𝑥/𝑦 ∴ ∆𝑧 = 𝑧 ( + ) = 2 ( + ) = 0.125 direction of the resultant and called rectangular form. The
𝑥 𝑦 8 4
𝒛 = 𝟐 ± 𝟎. 𝟏𝟐𝟓 value of 𝒊̂ indicates the magnitude in horizontal direction
(positive value of 𝒊̂ means Eastward or towards the direction of
Using nonconservative error estimate: the positive x-axis) and the value of 𝒋̂̂ indicates the magnitude in
vertical direction (positive value of 𝒋̂̂ means upward or towards the
1. 𝑧 = 𝑥 + 𝑦 ∴ ∆𝑧 = √∆𝑥2 + ∆𝑦2 = direction of the positive y-axis).
√(0.1𝑚)2 + (0.2𝑚)2 = 0.224 𝑚
Vector in Polar form
𝒛 = 𝟏𝟐 ± 𝟎. 𝟐𝟐𝟒 𝒎
2. 𝑧 = − + 𝑦 ∴ ∆𝑧 = √∆𝑥2 + ∆𝑦2 = Let 𝑥𝑇 and 𝑦𝑇 be total of x and y components respectively,
√(0.1𝑚)2 + (0.2𝑚)2 = 0.224 𝑚
𝒛 = 𝟒 ± 𝟎. 𝟐𝟐𝟒 𝒎 𝑀 = √𝑥𝑇2 + 𝑦𝑇2 = √(−5𝑘𝑚)2 + (−5 𝑘𝑚)2 = 𝟕. 𝟎𝟕𝟏 𝒌𝒎
3. 𝑧 = 𝑥𝑦 ∴ ∆𝑧 = 𝒛√( ∆𝒙 ∆𝒚 𝟐 𝒙𝑻 𝒚𝑻 Additional angle
𝟐 +( ) =
) 𝒚 + + 0°
𝒙
- + 180°
𝟑𝟐√( 𝟎.𝟏 𝟐
- - 180°
𝟐 𝟎.𝟐 = 1.649 𝑚2 + - 360°
) + ( 𝟒)
𝟖
𝒛 = 𝟑𝟐 ± 𝟏. 𝟔𝟒𝟗 𝒎𝟐 𝑦𝑇 −5𝑘𝑚
4. √ ∆𝒙 𝟐 ∆𝒚 𝟐 𝜃 = 𝑡𝑎𝑛−1 ( ) = 𝑡𝑎𝑛−1 ( ) = 45° + 𝟏𝟖𝟎° = 𝟐𝟐𝟓°
𝑧 = 𝑥/𝑦 ∴ ∆𝑧 = 𝒛 ( ) +( ) =
𝒙 𝒚 𝑥𝑇 −5𝑘𝑚
𝟐√ ( 𝟐 𝟐
𝟎.𝟏 𝟎.𝟐 = 0.103 𝑹 = 𝑴∠𝜽 = 𝟕. 𝟎𝟕𝟏∠𝟐𝟐𝟓° 𝒌𝒎
𝟖 ) + ( 𝟒)
𝒛 = 𝟐 ± 𝟎. 𝟏𝟎𝟑 Addition of vectors using Parallelogram Method
Vector and scalar quantities Vectors should be connected tail- to-
Vectors are quantities that are described with both magnitude and tail. From this, identical but
direction while scalars quantities are describe with magnitude imaginary vectors are drawn on the
alone. opposite sides of each vector (from
the head of the other vector). The
Some examples of scalar quantities are mass, length, and speed distance and direction from the tail-
while some examples of vector quantities are weight, to-tail connection of the two original
displacement, and velocity. vectors to the head-to-head
connection of the imaginary vectors are the magnitude and
Addition of vectors
direction of the resultant vector.
Since vectors have both magnitude and direction, addition of it is
Addition of vectors using Polygon Method
not as simple as addition of scalar. Direction of magnitude also
holds an important role. To easily execute the addition of vector, Vectors should be connected head-
finding its components will be the first step. to-tail (order does not matter). The
distance and direction from the tail
Direction of vectors can be written with the help of the 4 primary
of the first vector to the head of the
directions (North, East, West, and South) but it can also be
last vector are the magnitude and
represented by an angle. This angle is the angle between the vector
direction of the resultant vector.
and the positive x-axis rotating counterclockwise. To understand
this better, here is an example for different way of computation.
2|Page
a. Accurate and not precise
Activity 1 b. Accurate and precise
Direction: Convert the following given quantities to the c. Inaccurate and not precise
desired quantities. Show your solution: Express your d. Inaccurate and precise.
answer in standard form of scientific notation.
7. Three different people weigh a standard mass of 8.00 g on
the same balance. Each person obtains a reading of exactly
1. How many milligrams are there in 10 kilograms?
15.32 g for the mass of the standard. These results imply that
2. How many centimeters are there in 25
the balance that was used is:
hectometers?
a. precise but not accurate
3. How many pounds are there in 121 decigrams?
b. accurate but not precise
4. How many micrograms are there in 12.3
c. neither accurate nor precise
nanograms?
d. both accurate and precise
5. How may deciliters are there in 2.3 gallons?
8. In the table below, which group of data are considered the
6. How many seconds are there in 1.00 week?
most precise?
7. How many kg does a 145lb man weigh?
8. Annie is 5ft 4in tall. What is her height in meters?
a. Group 1
9. What is the equivalent of the density of
b. Group 2
Aluminum, 2.7g/cm3 in kg/m3?
c. Group 3
10. 10. A pressure of 1atm is equal to 1033g/cm2.
d. Group 4
Express this pressure in Lb. /in2.
9. Reymart, a Grade 12 student, performed an analysis of a
Activity 2
sample for its calcium content and got the following results:
12.91%, 12.88%, 12.91% and 12.89%. the actual amount of
Direction: Multiple Choice. Choose the letter of the correct calcium in the sample is 12.90%. What conclusions can you
answer . draw about the accuracy and precision of the obtained results?
1. Rowena conducts an experiment five times and gets a a. While precise, these results are not accurate
solution concentration of 3.1M, 2.9M, 2.8M, 3.0 M, and 2.2.9 b. While accurate, these results are not accurate
M. The known concentration of the solution is 2.0M. Which c. These results are both accurate and precise
of the following is true about Susan's results? d. These results are neither accurate nor precise
a. They are precise, but not accurate
b. They are accurate, but not precise. Activity 3
c. They are both accurate and precise Direction: Categorize the following statements into random
d. They are neither accurate nor precise errors and systematic errors. Write the correct code in the
table below.
2. Reynald is conducting an experiment. His first test gives
him a yield of 6.7 grams. His second test gives him a yield of C01 – unpredictable
1.8 grams. His third test gives him a yield of 4.5 grams. On C02 – produced consistent errors C03 - fixed amount of error
C04 – cannot be replicated
C05 – same error on repeated experiments
For number 6 to 10, show how to report the answer in the
average, his yield is 4.33 grams, which is close to the known following problems.
yield of 4.5 grams of substance. Which of the following is
true? Mass of Object A is = 44 ± 2 Mass of Object B is = 54 ± 3
a. His results are accurate but not precise Required: Total mass of the two objects.
b. His results are precise but not accurate
c. His results are both accurate and precise
d. His results are neither accurate nor precise
Activity 4
3. Dave is practicing for a golf tournament. His normal driver Direction: Write S if the quantity is scalar and V if it is a
distance is 200 yards. He hits three balls with his driver, and vector.
they travel a distance of 191 yards, 135 yards, and 189 yards. 1.Velocity
Which of the following is true? 2.Mass
a. His drives are accurate but not precise 3.Displacement
b. His drives are precise but not accurate 4.Density
c. His drives are both accurate and precise.
5.Work
d. His drives are neither accurate nor precise.
6.Volume
4. When a measurement is repeatable and consistent it is said
to have... 7.Acceleration
a. High precision 8.Force
b. Low precision 9.Distance
c. High accuracy 10.Time
d. Low accuracy
5. It is the description of how close a measurement is to the Activity 5
true value of the quantity measured. A. Addition of vectors by Parallelogram Method
a. accuracy b. precision c. estimates d. significant 1.By parallelogram method, find the resultant R of : M = 110
6. Looking at the rifle target, how would you describe the km 45o above x axis, N = 120 km 45o above -x axis.
shooting of the contestant in a dart competition?
3|Page
Direction: Multiple Choice: Write the letter of the correct
2.Find the resultant vector of the following displacements S1 answer on the space before the number.
= 7 km E, S2 = 9 km 45o N of E
1. Which graph of distance d against time t shows constant
B. Addition of vectors by Polygon Method speed?
1. Draw the following vectors head to tail to find the resultant
(Use appropriate scale)
A = 10 000 Newton west
B = 10 000 N south
C = 15 000 N 32o south of West D = 10 000 N 70o North of
East E = 20 000 N East
F = 15 000 N 15o South of East

C. Addition of vectors by Analytical/Mathematical


Method

1.Find the vector sum of the given vectors by completing the


table below. Express all the entries to the table in 4 decimal
places. Express θR in degrees-minutes- seconds (0 ‘ “)
2. A body travels 5 m , 10 m 15 m and 20 m in four successive
seconds. Its motion is said to be
a. uniform
b. irregular
c. accelerated
d. decelerated

3. An object travels with a constant speed of 20 m/s. after 2 s,


its speed is
a. doubles
b. triples
c. stays the same
d. decreased by ½

4. Which is true for a freely falling body?


a. Its speed is decreasing
b. Its speed is increasing
c. Its acceleration is decreasing
2.Four forces act simultaneously on an object. Find the d. Its acceleration is increasing
resultant force by component method.
5. Which of the following shows deceleration?
A = 75 dyne 20o N of E B = 95 a. A car moving 60 km/h in a straight path
dyne 70o N of E C = 55 dyne b. A bus moving 30 km/h around a curve
85o N of W D = 100 dyne 40o c. A typhoon moving at 30 km/h northwest and changing its
N of E path to 45 km/h west
d. A jeepney moving on a straight road, slowing down to a stop

Activity 6
Republic of the Philippines vice versa; solve for unknown quantities in equations involving
Department of Education Region
one-dimensional uniformly accelerated motion, including free
III – Central Luzon
SCHOOLS DIVISION OF CITY OF BALANGA BATAAN fall motion; and solve problems involving one-dimensional
NATIONAL HIGH SCHOOL – SENIOR HIGH SCHOOL motion with constant acceleration in contexts such as, but not
City of Balanga, Bataan limited to, the “tail-gating phenomenon”, pursuit, rocket
launch, and freefall problems)
ACTIVITY SHEETS IN GENERAL PHYSICS 1
Acceleration is the rate of change of velocity in each unit of time
Quarter 1 - Week 2, Day 1- 4 (Convert a verbal description of a (m/s2). It answers the question “how fast the velocity changes?”
physical situation involving uniform acceleration in one
Velocity is the distance traveled in each unit of time with direction
dimension into a mathematical description; interpret
(m/s). It answers the question “how fast a system moves?”
displacement and velocity, respectively, as areas under velocity
vs. time and acceleration vs. time curves; interpret velocity and The following graphs show the motion of a system with different
acceleration, respectively, as slopes of position vs. time and representation
velocity vs. time curves; construct velocity vs. time and
acceleration vs. time graphs, respectively, corresponding to a Displacement vs time (constant velocity)
given position vs. time-graph and velocity vs. time graph and
4|Page
This graph reflects the mathematical equation for this is as follow:
motion of the system for a 1
given time. The slope d=∫ f ( t )=∫ (C +mt )∙ dt =¿ ∫ ( v i+ at ) ∙ dt =v i t+ at 2 ¿.
2
represents the velocity (
Notice a small change in the equation since the initial velocity is not
rise d
m= = =v ). The zero.
run t
function f(t) tells the Displacement d is also equal to the change in position or ∆ P .
displacement of the system a 1 1
∆ P=d=P f −Pi=v i t+ a t 2 ∴ P f =P i+ v i t + a t 2. Other
certain time ( f ( t )=d=v ∙ t 2 2
). A linear relation between the displacement and time is clearly kinematic equations can be derived from this equation. The
shown too therefore, the system’s velocity is constant, or the motion following situation is an example.
is said to be uniform.
A rock is drop from a certain height. It took 5 s before hitting the
Velocity vs time (constant ground. Ignoring air resistance, how high is the initial position of
acceleration) the rock?
This graph shows the velocity m
vs time graph of a system’s a=g=−9.8 (acceleration due to gravity, downward)
motion. The slope of this s2
graph represents the
acceleration of the system ( m
vi =0 (no force acted on the system except gravity)
rise v ). A linear s
m= = =a
run t t=5 s (time it took before it hit the ground)
relationship between velocity
and time is also shown which Pf =0 m(final position is at the ground)
means that the system’s acceleration is constant however, velocity
is changing. Other quantities of the system can also be derived from Pi=? (unknown quantity)
this graph. Area under this function is the displacement.
1 Pi
d=∫ f ( t )=∫ at ∙ dt= a t 2 Deriving the formula for
2
1 1
Pf =Pi +v i t+ g t 2 Pi=P f −v i t− g t 2. The first two
2 2
terms have been canceled since Pf and vi are both zero.
−1 2 −1 m

Acceleration vs time
∴ P i=
2
gt =
2 ( s )
−9.8 2 ( 5 s )2=122.5 m

(constant acceleration) Other kinematic equation


Since acceleration is v i+ v f
2 d v =v +at
constant, it does not change
as time changes. The area
under acceleration vs. time
t=
√ a
; f i ; d=
2
2 2
∗t ; and v f =v i +2 ad

graph is the velocity of the


system.

∫ f ( t ) =∫ ( a+0 t ) ∙ dt =at=v .
The relationship between graphs is shown making the construction
of one graph from the other possible.

Kinematics

Motion along a straight line

A system moving along a straight line have quantities associated


with it. These quantities are but not limited to displacement d ,
velocity v , acceleration a , initial position Pi, and final position
Pf .
Graph shown beside is the
graphical representation of
a system in a constant
acceleration. Unlike the
first velocity vs time
graph, the velocity on this
graph does not start at
zero. The area under this
function is still the
displacement of the
system within a given period of time (as shown in the graph). The
5|Page
Activities Direction: The following question is referring to the concept of
solving problems involving one-dimensional motion with
Activity 1 constant acceleration. Answer the following questions solve for
the unknown quantities. Round-off your final answers in the
Direction: The following question is referring to the concept of
nearest hundredths if computation is required.
interpretation on velocity and acceleration as slopes of position
vs. time and velocity vs. time curves respectively. Answer the 1. A motorist traveling with a constant speed of about 15 m/s
following questions to interpret both using a graph. Round-off passes a school-crossing corner, where the speed limit is
your final answers in the nearest hundredths if computation is 10 m/s. Just as the motorist passes the school-crossing
required. sign, a police officer on a motorcycle stopped there starts
in pursuit with a constant acceleration of 3.0 m/s2. (a)How
1.A bus going to Balanga leave the terminal and had a 40 km/h
much time elapses before the officer passes the motorist?
velocity within the first 10s of travel. (a)Graph the velocity vs time
(b) What is the officer’s speed at that time? (c) At that
of the bus. (b)Find the acceleration of the bus by finding the slope
time, what distance has each vehicle traveled?
of the graph.

2.If the bus on situation number 1 decided to stop accelerating after


Republic of the Philippines
5 more seconds and had a constant velocity, (a)Graph its position Department of Education Region
vs. time (assumes that the position is in positive x-axis) in the next III – Central Luzon
10 more seconds. (b) Find its velocity by finding the slope of the SCHOOLS DIVISION OF CITY OF BALANGA BATAAN
graph. NATIONAL HIGH SCHOOL – SENIOR HIGH SCHOOL
City of Balanga, Bataan
Activity 2
ACTIVITY SHEETS IN GENERAL PHYSICS 1
Direction: The following question is referring to the concept of
constructing velocity vs. time and acceleration vs time graphs
Quarter 1 - Week 3, Day 1- 4 (Describe motion using the
from a given position vs. time graphs and velocity vs. time
concept of relative velocities in 1D and 2D; deduce the
graphs, respectively and vice versa. Answer the following
consequences of the independence of vertical and horizontal
questions to construct both graphs. Round-off your final
components of projectile motion; calculate range, time of flight,
answers in the nearest hundredths if computation is required.
and maximum heights of projectiles; infer quantities associated
1.Figure on the right shows the velocity vs. time of a cat walking with circular motion such as tangential velocity, centripetal
along the positive x- axis. (a)Find the cat’s velocity at t=0 to t=6 acceleration, tangential acceleration, radius of curvature; and
with 1 second interval. (b) Find the cat’s acceleration by finding the Solve problems involving two dimensional motion in contexts
slope of the graph. (c) Construct an acceleration vs. time graph such as, but not limited to ledge jumping, movie stunts,
using the given velocity vs. time graph. (d) Construct a position vs. basketball, safe locations during firework displays, and Ferris
time graph using the given velocity vs. time graph. wheels)

Activity 3 Relative velocity is simply the velocity of one object in reference to


the other (both objects are often in motion). The following situation
is an example..

Bus A is travelling 60 km/h due east while Bus B is travelling 40


km/h due east. The relative velocity of Bus A with respect to Bus B
is V AB=V A−V B=60 km/h−40 km/h=20 km/h and the
relative velocity of Bus B with respect to Bus A is
V BA =V B−V A =40 km/h−60 km/h=−20 km/h.
Projectile motion is the curve motion of the system along
horizontal and vertical axis and is affected by gravity. Although
projectile motion is a combination of horizontal motion and vertical
motion, these two components are independent. This means that the
motion vertically in not affected by motion horizontally. Some
quantities associated with projectile motions are but not limited to
time of flight t , range R , and maximum height h max. For the sake
Direction: The following question is referring to the concept of of the simplicity of the discussion, the system in all projectile
solving the unknown quantities in equation involving free fall motion in this section does not bounce.
motion. Answer the following questions solve for the unknown
Time of flight t is the time the system takes from its initial position
quantities. Round-off your final answers in the nearest
hundredths if computation is required. to its final position.
Range R is the horizontal distance from its initial position to its
final position.
Maximum height h max is already self-explained. It is the maximum
1.You were on the 4th floor of a 4-story building and decided to height of the projectile motion.
determine the height of you position from the ground. You have Below is an example of a projectile motion.
limited resources but have a rock massive enough to neglect air A ball is launch with an initial velocity of 10 m/s 15° above the
resistance and a super accurate timer. You then decided to let go of horizontal line from an initial height of 10 m. Find the (a) time of
the rock from your position down to the ground. Using your super
flight t , (b) range R , and (c) maximum height h max of the projectile
accurate timer, you manage to time its flight and it took exactly
1.874 s before hitting the ground. (a) Calculate the height from the motion.
ground to your position.

(b) Assumes that the height of each story of the building is


even, find the height of the building from the ground to the 4th
floor’s ceiling.

Activity 4
6|Page
Derivation of quantities associated with circular motion
Tangential Velocity v tan
The distance an object traveled
along a circular path in a
complete cycle is equivalent to
its circumference (2 πR ) and
the time it takes for one cycle
is called period (T ). This
2 πR
means that v tan =
T
Centripetal Acceleration
a rad
Since vertical and horizontal motion are independent to one |∆ v| ∆ s or v
another, x and y components are needed to extract from the given = |∆ v|= 1 ∆ S
vector first. v1 R R
x−componnet =Vcos ( θ )=10cos ( 15 )=9.659 m/s Absolute value is used since
we are just dealing with
y−componnet=Vsin ( θ )=10 sin ( 15 )=2.588 m/s magnitude.
(a) time of flight is computed from the vector’s vertical motion Average acceleration is the
1 change of velocity per unit of
Pf =Pi +v i t+ at 2, to its general
Rearranging this equation,
2 time.
1 2 |∆ v| v 1 ∆ s
form ( A x 2+ Bx+ C=0), a t + v i t + Pi −P f =0 . To find the a av = =
2 ∆t R ∆t
roots of quadratic equation, using a quadratic formula is required
thus,
2
−B ± √ B 2−4 AC −v i ± v i −4 ( 0.5 ) (−9.8 ) ( Pi−Pf ) .

t= =
2A −9.8
2
−2.588 ± √ 2.588 −4 ( 0.5 ) (−9.8 )( 10−0 )
t=
−9.8
t 1=−1.189 s∧t 2=1.717 s
Time doesn’t run backward; therefore, no time is written in negative
value. The time of flight of the projectile motion is t=1.717 s.
Time of flight are often the negative square root of the equation. ∆s v2 4 π 2 R
(b) range R or horizontal distance are calculated from the motions =v ∴ arad = =
∆t R T2
horizontal motion.
1
R=P f =P i+ v i t + a t 2. Remember that during the horizontal Example:
2 Passengers on a carnival ride move at constant speed in a horizontal
motion, to other forces are acting on the system. Notice also that circle of radius 5.0 m, making a complete circle in 4.0 s. What is
range is just the horizontal distance from initial position to its final their (a) tangential velocity v tan and (b) centrepetal acceleration
position. With these, the horizontal motion’s acceleration and initial a rad.
position are both zero. Range now is equal to
R=v i t=9.659 m/s ∙ 1.717 s=16.585 m. 2 πR 2 π ( 5.0 m )
(a) v tan= = =7.854 m/s
(c) Maximum height can also be calculated from its vertical motion. T 4s
2
During its peak height, its vertical velocity is zero. Vertical velocity v 2 ( 7.854 m/ s )
changes due to the acceleration brought by Earth’s gravity. (b) a rad = = =12.337 m/s 2
R 5.0 m
v f −v i v f −vi 0−2.588 m/s Using different method
a= ∴ t= = =0.264 s . This is the
t a −9.8 m/ s2 2
4 π 2 R 4 π (5.0 m)
time it takes before the system reached its peak height. Maximum (b) a rad = 2
= 2
=12.337 m/s 2
1 2 T (4.0 s)
height now is h max=P f =P i + v i t+
at .
2
m 1 m Activities
h max=10 m+2.588 ( 0.264 s )+ −9.8
s 2 s ( )
( 0.264 s )2=10.342 m
Activity 1
Circular Motion
There are two types of circular motion namely uniform circular Direction: The following question is referring to the concept of
motion and nonuniform circular motion. Uniform circular motions relative velocities in 1D and 2D, answer the following questions to
are motion in a circular path with constant speed while nonuniform describe what it is. Round-off your final answers in the nearest
circular motions are the opposite. hundredths if computation is required.
Radius of curvature R is the radius or distance of the circular path
1. A man is sitting on a train with a constant velocity of 10 m/s
to its center. A circular motion does not have to complete a circular
East. The man then stands up and walk backward with a velocity
path to have a radius of curvature.
of 2 m/s. (a) Did the man’s velocity with respect to the Earth
Tangential velocity v tan is the velocity tangent to the circular change? (b) If yes, did it become faster or slower? (c) calculate
motion. Since velocity has a direction and circular motion always the relative velocity of the man with respect to the Earth.
change direction, velocity can’t be used to describe its motion.
Centripetal acceleration a rad is the towards the center of the 2. A group of men decided to cross a river with a motorboat. The
circular path. Responsible for the change of direction of tangential average velocity of the river’s flow is 3 m/s. If the boat is
velocity. traveling at a velocity of 4 m/s North, (a) will the boat lands on
7|Page
exact opposite side of the river where they started? (b) what it force but the ground pushes him upward with equal but opposite
the boat’s relative velocity with respect to the Earth? (c) what is force. This is also the action-reaction pair. The action force is the
the boat’s relative velocity with respect to the river. gravitational force while the grounds push is the reactions force.
2nd law states that when a net force F is applied on a system with a
Activity 2 mass m, it will start to accelerate a. F=ma .
Direction: The following questions are referring to the concept 3rd law states in every action, there is equal but opposite reaction.
and components of projectile motion, answer the following Contact forces are forces that needs physical contact before
questions to describe what it is. Round- off your final answers in affecting the system. Some examples are push, pull, tension and
the nearest hundredths if computation is required. compression.
Noncontact forces are forces that doesn’t need physical contact to
take an effect on the system. Some examples are gravitational pull
and magnetism.
1. Two balls were released at the same height and time. Ball A is Four common types of force:
pushed sideward while Ball B was just dropped. Which of the
two balls will reached the ground first? (Explain your answer).

2. A ball was kicked from the top of a 150 m cliff generating an


initial velocity of 20 m/s with an angle 10° above the horizontal
(as shown on the figure). Find (a) the time it takes before hitting
the ground and (b) the distance it traveled from its initial
horizontal position to the place of impact.

Activity 3

Direction: The following questions are referring to the concept of


circular motions, answer the following questions to help
understand the quantities associated with it. Round-off your final
answers in the nearest hundredths if computation is required.

1. A jet is flying at 134.1 m/s along a straight line and makes a turn
along a circular path level with the ground. What does the radius
of the circle have to be to produce a centripetal acceleration of 1
g on the pilot and jet toward the center of the circular trajectory?

2. A flywheel has a radius of 20.0 cm. What is the speed of a point


on the edge of the flywheel if it experiences a centripetal
acceleration of 900.0 cm/s2?
Friction is a force between the surface and the object and is always
Activity 3
on the opposite direction.
Direction: Answer the following questions. Round-off your final Static friction F s is a frictional force present when an object is not
answers in the nearest hundredths if computation is required. yet in motion. F s=n ∙ μs where n is normal force and μs is static
frictional coefficient. This coefficient depends on the materials
1. A 900-kg car moving at 10 m/s takes a turn around a circle with
being rub against each other.
a radius of 25.0 m. Determine the acceleration and the net force
acting upon the car. Kinetic friction F k is a frictional force present when an object is
already in motion. F k =n ∙ μ k where n is normal force and μk is
2. A 95-kg halfback makes a turn on the football field. The kinetic frictional coefficient. This coefficient depends on the
halfback sweeps out a path that is a portion of a circle with a materials being rub against each other.
radius of 12-meters. The halfback makes a quarter of a turn Example:
around the circle in 2.1 seconds. Determine the speed, An object with a mass of 2kg is
acceleration and net force acting upon the halfback. place on a ramp with an angle
θ=38.66 ° (as shown in the
figure). The static friction
Republic of the Philippines coefficient and kinetic friction
Department of Education
Region III – Central Luzon coefficient between the two
SCHOOLS DIVISION OF CITY OF BALANGA objects are 0.8 and 0.5
BATAAN NATIONAL HIGH SCHOOL – SENIOR HIGH SCHOOL respectively. Assumed that the object does not move, (a) draw it
City of Balanga, Bataan free-body diagram and find its (b) weight, (c) normal force, and (d)
static friction. (e) Check if the calculated values are correct if the
Quarter 1 - Week 4, Day 1- 4 (Define inertial frames of system are in equilibrium.
reference; identify action-reaction pairs; draw free-body
(a) (b)
diagrams; apply Newton’s 1st law to obtain quantitative and
qualitative conclusions about the contact and noncontact forces Weight F w =mg
acting on a body in equilibrium; and differentiate the properties m
of static friction and kinetic friction) (
F w =2 kg 9.8
s )
Internal frame of reference is a frame of reference in which ¿ 19.6 N
Newton’s first law of motion is applicable. This frame can be in
motion or at rest.

Newton’s Law of Motion (c) Normal force n=F w ∙cos ( θ )


1st law states that a system in motion will remain in motion and a ¿ 19.6 N ∙ cos ( 38.66 ° )=15.305 N
system at rest will remain at rest if no net force is applied.
(d) Static friction F s =n ∙ μ s
A body at rest does not mean that no force is acting on it. A man
standing on the ground for example, experiences a gravitational ¿ 15.305 N ∙ 0.8=12.244 N
8|Page
(e) To check if the calculated values are correct in a equilibrium, the mind, how will you differentiate the properties of kinetic friction
net force in both horizontal and vertical should be zero. and static friction?
Summation of forces vertical taking upward as positive
2.You want to move a 500-N crate across a level floor. To start the
∑ F v =n∙ sin ( 51.34 ) + F s ∙sin ( 38.66 )−F w=0 crate moving, you have to pull with a 230-N horizontal force. Once
¿ 15.305 N sin ( 51.34 )+ 12.244 N sin ( 38.66 )−19.6 N the crate “breaks loose” and starts to move, you can keep it moving
¿ 11.951 N +7.649 N −19.6 N=0 at constant velocity with only 200 N. What are the coefficients of
Summation of forces horizontal taking to the right as positive static and kinetic friction?
∑ F H =−n ∙ cos ( 51.34 ) + F s ∙cos ( 38.66 )=0
¿−15.305 N ∙ cos ( 51.34 ) +12.244 N ∙ cos ( 38.66 )=0 Republic of the Philippines
Department of Education
¿−9.561 N + 9.561 N =0 Region III – Central Luzon
SCHOOLS DIVISION OF CITY OF BALANGA
Activities BATAAN NATIONAL HIGH SCHOOL – SENIOR HIGH SCHOOL
City of Balanga, Bataan
Activity 1

Direction: Answer the following questions. Round-off your final Quarter 1 - Week 5, Day 1- 4 (Apply Newton’s 2nd law and
answers in the nearest hundredths if computation is required. kinematics to obtain quantitative and qualitative conclusions
about the velocity and acceleration of one or
1.A 900-kg car moving at 10 m/s takes a turn around a circle with a more bodies, and the contact and noncontact forces acting on
radius of 25.0 m. Determine the acceleration and the net force one or more bodies; Solve problems using Newton’s Laws of
acting upon the car. motion in contexts such as, but not limited to, ropes and pulleys,
2.A 95-kg halfback makes a turn on the football field. The halfback the design of mobile sculptures,
sweeps out a path that is a portion of a circle with a radius of 12- transport of loads on conveyor belts, force needed to move
meters. The halfback makes a quarter of a turn around the circle in stalled vehicles, determination of safe driving speeds on banked
2.1 seconds. Determine the speed, acceleration and net force acting curved roads; Calculate the dot or scalar product of vectors;
upon the halfback. Determine the work done by a force acting on a
system)
Activity 2
Newton’s 2nd law states if a net external force acts on a body, the
Direction: Answer the following questions. Round-off your final body accelerates. The direction of the acceleration is same as the
answers in the nearest hundredths if computation is required.
direction of the net force. Σ F=ma
1.You were riding a fully air-conditioned bus (all windows and doors
are closed) from Mariveles to Balanga and it has a constant Example:
velocity. If you let a balloon float inside the bus, (a) will it float An iceboat is at rest on a frictionless horizontal surface. A wind is
backward, forward or just straight up? Explain your answer.
blowing along the direction of the runners so that 4.0 s after the
2.You put a book on top of a level table. Assumes that the table is iceboat is released, it is moving at (about or). What constant
stable before and after you put the book. (a) Will the book move? horizontal force does the wind exert on the iceboat? The combined
(b) What forces are present in the book? (c)Are all the forces in the mass of iceboat and rider is 200 kg.
book equal in magnitude but opposite in direction?

Activity 3

Direction: The following questions are referring to the concept of


free-body diagram and Newton’s 1st law, answer the following
questions to qualitative and quantitative conclusions. Round-off
your final answers in the nearest hundredths if computation is
required.

1. A ball with a mass of 10 kg is hanging from a long string that is The iceboat starts at rest (its initial x-velocity is v0x = 0) and it attains
tied to the ceiling of a train car traveling eastward on horizontal an x-velocity vx= 6.0 m/s after an elapsed time t= 4.0s. To relate the
tracks. An observer inside the train car sees the ball hang x-acceleration ax to these quantities we use
motionless. Draw a clearly labeled free-body diagram for the ball if
(a) the train has a uniform velocity, and (b) the train is speeding up vx=v0x + axt. There is no vertical acceleration, so we expect that the
uniformly. (c) Is the net force on the ball zero in either case? normal force on the iceboat is equal in magnitude to the iceboat’s
Explain. (d) If the train stops from 60 km/h velocity within 5 s, weight.
draw a clearly labeled free-body diagram for the ball and compute
for the tensional force on the string (assumes that the string didn’t Known quantities: Unknown quantities:
break). m= 200kg ax= acceleration
vax= 0 (initial velocity) N= normal force
Activity 3 vx= 6.0 m/s (final velocity) Fw= horizontal force
t= 4.0s
Direction: The following questions are referring to the concept of
static and kinetic friction, answer the following questions to Equations:
differentiate the two. Round-off your final answers in the nearest
hundredths if computation is required. Σ Fx = Fw = max
1.Imagine you are pushing a wooden object along a level wooden Σ Fy = n + (-mg) = 0 so n = mg
ground. The object has a cubic shape and much easier to push rather
than to roll. You notice that it is always harder to push at the Vx= v0x + axt
beginning but when the object starts to move, it only requires lesser
amount of force. Aside from inertia, you remembered 1 more factor To find we first solve this third equation for and then
which is the static and kinetic friction. Static friction present when substitute the result into the equation:
an object is not in motion while kinetic is the opposite. With this in
9|Page = 1.5 m/s2
ax = vx - v0x 6.0 m/s - 0 m/s hand rule. Two vectors to be multiplied and the product must be
=
t 4.0 s perpendicular to each other.

FW = max = (200 kg) (1.5 m/s2) 2 = 300 kg m/s2

Activities

Since 1 kg m/s2 = 1N, the final answer is Activity 1


FW = 300 N (about 67 lb)
Direction: The following questions are referring to the concept of
Newton’s 2nd law and kinematics, answer it to obtain
Tension: force T = a pulling force exerted on an object by a rope, quantitative and qualitative conclusions about it. Round-off your
cord etc. final answers in the nearest hundredths if computation is
required.
T = w + may = mg + may = m (g + ay)
1. An average person can reach a maximum height of about 60 cm
when jumping straight up from a crouched position. During the
jump itself, the person’s body from the knees up typically rises a
distance of around 60 cm. To keep the calculations simple and
Example: yet get a reasonable result, assume that the entire body rises this
much during the jump. (a) With what initial speed does the
An elevator and its load have a person leave the ground to reach a height of 60 cm? (b) Draw a
combined mass of 800 kg. The free-body diagram of the person during the jump. (c) In terms of
elevator is initially moving this jumper’s weight w, what force does the ground exert on him
downward at 10.0 m/s; it slows to or her during the jump?
a stop with constant acceleration in
a distance of 25.0 m. What is the Activity 2
tension T in the supporting cable
Direction: The following questions are referring to the
while the elevator is being brought to rest?
concept of Newton’s law of motion, answer it to understand
better. Round-off your final answers in the nearest
The elevator is moving in the negative y-direction, so its initial
hundredths if computation is required.
y-velocity v0y and its y-displacement y-y0 are both negative:
v0y = -10.0m/s and y-y0 = -25.0 m. The final y-velocity is vy =0 1. A 15.0-kg load of bricks hangs
from one end of a rope that
Known quantities: Unknown quantities: passes over a small, frictionless
m = 800 kg ay= y-acceleration pulley. A 28.0 kg counterweight
v0y = -10 m/s (downward) T= Tension is suspended from the other end
y0 = 25.0 m of the rope, as shown in the
figure beside. The system is
Equations: released from rest. (a) Draw two
free-body diagrams, one for the
load of bricks and one for the
Σ Fy = T + (-w) = may counterweight. (b) What is the
T = w + may = mg + may = m (g + ay)
magnitude of the upward
acceleration of the load of
To determine ay, rewrite the constant-acceleration equation:
bricks? (c) What is the tension
vy2 = v0y2 + 2ay (y - y0) to: 2 2 in the rope while the load is
ay = vy – v0y moving? How does the tension
2 (y-y0)
compare to the weight of the load of bricks? To the weight
Transpose: of the counterweight?
(0)2 – (-10.0 m/s)2
ay = = +2.00 m/s2
2 (-25.0 m) Activity 3

We will use T= m (g + ay) in getting the Tension. Direction: The following questions are referring on
calculation of the dot or scalar product of vectors, answer the
T= (800 kg)(9.80 m/s2 + 2.00 m/s2) = 9440N following questions to determine the work done by a force.
Round-off your final answers in the nearest hundredths if
computation is required.
Dot product and Cross product
1.A tow truck pulls a car 5.00 km along a horizontal roadway
Dot product is one of the products of vectors. Although two using a cable having a tension of 850 N. (a) How much work
vectors were multiplied to each other, product of dot product is does the cable do on the car if it pulls horizontally? If it pulls at
35° above the horizontal? (b) How much work does the cable do
scalar. This is also called scalar product. ( A ∙ B=AB ∙ cos ( θ ) )
on the tow truck in both cases of part (a)? (c) How much work
does gravity do on the car in part (a)?
Cross product, on the other hand, is a product of two vectors
which is also a vector. This
means that the product here
has its direction
Buoyancy and Archimedes’ Principle
( A X B= A X B ∙sin ( θ ) ).
The direction of the product is Buoyancy is the force exerted on an object that is wholly or partly
determine using the right- immersed in a fluid.

10 | P a g
e
Archimedes’ principle states that the upward buoyant force that is Whenever a new quantity is introduced in physics, the standard
exerted on a body immersed in a fluid, whether fully or partially metric units associated with that quantity are discussed. In the case
submerged, is equal to the weight of the fluid that the body of work (and also energy), the standard metric unit is the Joule
displaces. Archimedes' principle is a law of physics fundamental to (abbreviated J). One Joule is equivalent to one Newton of force
fluid mechanics. causing a displacement of one meter. In other words,
Republic of the Philippines
Department of Education
Region III – Central Luzon
SCHOOLS DIVISION OF CITY OF BALANGA
BATAAN NATIONAL HIGH SCHOOL – SENIOR HIGH SCHOOL
City of Balanga, Bataan

Sample Problem
Quarter 1 - Week 6, Day 1- 4 (Interpret the work done by a 1. Ben Travlun carries a 200-N suitcase up three flights of stairs (a
force in one- dimension asan area under a Force vs. Position height of 10.0 m) and then pushes it with a horizontal force of
curve, Relate the gravitational potential energy of a system or 50.0 N at a constant speed of 0.5 m/s for a horizontal distance of
object to the configuration of the system, Relate the elastic 35.0 meters. How much work does Ben do on his suitcase during
potential energy of a system or object to the configuration of the this entire motion?
system, Explain the properties and the effects of conservative
forces Solutions:
The motion has two parts: pulling vertically to displace the
suitcase vertically (angle = 0 degrees) and pushing horizontally
When a force acts upon an object to cause a displacement of the to displace the suitcase horizontally (angle = 0 degrees).
object, it is said that work was done upon the object. There are
three key ingredients to work - force, displacement, and cause. For For the vertical part,
a force to qualify as having done work on an object, there must be a W = (200 N) * (10 m) * cos (0 deg) = 2000 J.
displacement and the force must cause the displacement. There are
several good examples of work that can be observed in everyday For the horizontal part,
W = (50 N) * (35 m) * cos (0 deg) = 1750 J.
life - a horse pulling a plow through the field, a father pushing a
grocery cart down the aisle of a grocery store, a freshman lifting a The total work done is 3750 J (the sum of the two parts).
backpack full of books upon her shoulder, a weightlifter lifting a
barbell above his head, an Olympian launching the shot-put, etc. In 2. A force of 50 N acts on the block at the angle shown in the
each case described here there is a force exerted upon an object to diagram. The block moves a horizontal distance of 3.0 m. How
cause that object to be displaced. much work is done by the applied force?

Work Equation

Mathematically, work can be expressed by the following equation.

W = F • d • cos Θ

where F is the force, d is the displacement, and the angle (theta) is


defined as the angle between the force and the displacement vector.
Perhaps the most difficult aspect of the above equation is the angle
"theta." The angle is not just any 'ole angle, but rather a very Solutions:
specific angle. The angle measure is defined as the angle between
the force and the displacement. To gather an idea of its meaning, W = F * d * cos(Theta)
consider the following three scenarios.
W = (50 N) * (3 m) * cos (30 degrees) = 129.9 Joules
Scenario A: A force acts
3. Apply the work equation to determine the amount of work done
rightward upon an object as it is by the applied force in each of the three situations described
displaced rightward. In such an below.
instance, the force vector and the
displacement vector are in the
same direction. Thus, the angle
between F and d is 0 degrees.

Scenario B: A force acts leftward


upon an object that is displaced
rightward. In such an instance, the
force vector and the displacement
vector are in the opposite direction. Thus, the angle between F and
d is 180 degrees.

Scenario C: A force acts upward


on an object as it is displaced
rightward. In such an instance,
the force vector and the
displacement vector are at right
Solutions:
angles to each other. Thus, the angle between F and d is 90 degrees.
Diagram A Answer:
Units of Work
W = (100 N) * (5 m)* cos(0 degrees) = 500 J

11 | P a g
e
The force and the displacement are given in the problem statement. It en Travlun carries a 200-N suitcase up three flights of stairs (a
is said (or shown or implied) that the force and the displacement are height of 10.0 m) and then pushes it with a horizontal force of
both rightward. Since F and d are in the same direction,the angle is 0 50.0 N at a constant speed of 0.5 m/s for a horizontal distance of
degrees. 35.0 meters. How much work does Ben do on his suitcase during
this entire motion?
 
3. Calculate the work done by a 2.0-N force (directed at a 30° angle
Diagram B Answer: to the vertical) to move a 500 gram box a horizontal distance of
400 cm across a rough floor at a constant speed of 0.5 m/s.
W = (100 N) * (5 m) * cos(30 degrees) = 433 J
(HINT: Be cautious with the units.)
The force and the displacement are given in the problem statement. It
4. How much work is done by an applied force to lift a 15-Newton
is said that the displacement is rightward. It is shown that the force is
block 3.0 meters vertically at a constant speed?
30 degrees above the horizontal. Thus, the angle between F and d is
5. A student with a mass of 80.0 kg runs up three flights of stairs in
30 degrees.
12.0 sec. The student has gone a vertical distance of 8.0 m.
 Diagram C Answer: Determine the amount of work done by the student to elevate his
body to this height. Assume that his speed is constant.
W = (147 N) * (5 m) * cos(0 degrees) = 735 J
Republic of the Philippines
The displacement is given in the problem statement. The applied Department of Education
Region III – Central Luzon
force must be 147 N since the 15-kg mass (Fgrav=147 N) is lifted at SCHOOLS DIVISION OF CITY OF BALANGA
constant speed. Since F and d are in the same direction, the angle is 0 BATAAN NATIONAL HIGH SCHOOL – SENIOR HIGH SCHOOL
degrees. City of Balanga, Bataan

Activities
Quarter 1 - Week 7, Day 1- 4 (Use potential energy diagrams to
Activity 1. infer force; stable, unstable, and neutral equilibria; and turning
points; Solve problems involving work, energy, and power in
1. Before beginning its initial descent, a roller coaster car is always
contexts such as, but not limited to, bungee jumping, design of
pulled up the first hill to a high initial height. Work is done on
roller-coasters, number of people required to build structures
the car (usually by a chain) to achieve this initial height. A
such as the Great Pyramids and the rice terraces; power and
coaster designer is considering three different incline angles at
energy requirements of human activities such as sleeping vs.
which to drag the 2000-kg car train to the top of the 60-meter
sitting vs. standing, running vs. walking; Differentiate center of
high hill. In each case, the force applied to the car will be applied
mass and geometric center; and Relate the motion of center of
parallel to the hill. Her critical question is: which angle would
mass of a system to the momentum and net external force acting
require the most work? Analyze the data, determine the work
on the system)
done in each case, and answer this critical question.

2. Energy is the ability to do work and is equal to the force multiply


B by displacement parallel to it (W =Fd ∙ cos ⁡(θ)). Although
energy is scalar and calculated using dot product or scalar product
of two vectors, energy still has its direction. Not the same direction
as other vectors but energy can be determined whether it is
absorbed or released by the system. If net energy is positive, energy
is absorbed by the system and if net energy is negative, it is released
by the system.
Gravitational potential energy is energy at rest and due to gravity
(often due to Earth’s gravity). U g=mgh where; m is mass of the
object, g is acceleration due to gravity (9.8 m/s 2), and h is height
from the ground.
Kinetic energy is energy in motion. System’s kinetic energy also
1
depends on mass m . K= mv 2
2
Potential energy diagram is a graphical representation of the
system’s potential energy with respect to its position (often in x axis
in one dimension). Below is an example
This graph shows a hypothetical potential-energy function of a

system. The slope of the curve determines the state of the system
with respect to time. System always tends to move to a lower
potential energy. Energy at x 1 and x 3 is said to be in stable
equilibrium since slope at both sides tends toward a higher
12 | P a g
e
potential energy. Energy at x 2 and x 4 are both in unstable
equilibrium since both side of the curve tends to a lesser potential
energy. Notice that the slope of both equilibrium points is equal to
zero. If the total energy of the system is greater than the potential
energy at x 4 or E3 , the system will overcome the unstable
equilibrium point and continue towards the lower potential-energy
point.
Activities
Equilibrium is state when all forces acting on the system are
Activity 1

Describe the given diagram


and identify each dot
whether the potential
energy diagram is in stable,
unstable, and neutral
equilibria and give a short description.

Activity 2
Label each diagram whether stable, unstable and neutral
equilibrium.
balanced. The following are the three types of equilibrium.
1. Stable equilibrium is when the gravitational potential
energy increases as the system moves away from its
equilibrium position. This
means that when the system is
moved away from its
equilibrium position, it will
return (as long as the motion
does not exceed to its limit).
Motion is always toward to the
lower potential energy. Activity 3

Solve the following problems using your idea in energy


2. Unstable conservation, work, and power.
equilibrium is when gravitational
potential energy decreases as the system 1.Renata Gass is out with her friends. Misfortune occurs and Renata
moves away from its equilibrium position. and her friends find themselves getting a workout. They apply a
This means that when the system is cumulative force of 1080 N to push the car 218 m to the nearest fuel
slightly moved away from its equilibrium station. Determine the work done on the car.
position, the system will continue to move
with the help of gravity. 2.Hans Full is pulling on a rope to drag his backpack to school across
the ice. He pulls upwards and rightwards with a force of 22.9 Newton
at an angle of 35 degrees above the horizontal to drag his backpack a
3. Neutral equilibrium is when the horizontal distance of 129 meters to the right. Determine the work (in
system’s gravitational potential energy does not change as position Joules) done upon the backpack.
changes. This means that when the system is moved, it’s neither
return to its equilibrium position nor accelerates in its motion. 3.Lamar Gant, U.S. powerlifting star, became the first man to deadlift
five times his own body weight in 1985. Deadlifting involves raising
a loaded barbell from the floor to a position above the head with
Center of mass and geometric center outstretched arms. Determine the work done by Lamar in deadlifting
Center of gravity is the point at which the distribution of gravity is 300 kg to a height of 0.90 m above the ground.
equal to all direction and does depend on gravity. 4.Sheila has just arrived at the airport and is dragging her suitcase to
Center of mass is the point at which the distribution of mass is the luggage check-in desk. She pulls on the strap with a force of 190
equal to all direction and does not depend on gravity. It is also the N at an angle of 35° to the horizontal to displace it 45 m to the desk.
reference at which most forces and energy are concentrated. Determine the work done by Sheila on the suitcase.
C m−x =
∑ x i mi = ( x 1 m1 + x 2 m2 + x 3 m3 +… x n mn ) ( 5.While training for breeding season, a 380-gram male squirrel does
∑ mi (m 1+ m2 +m 3 …+m n) 32 pushups in a minute, displacing its center of mass by a distance of
along x−axis ¿ 8.5 cm for each pushup. Determine the total work done on the
∑ y i mi = ( y 1 m1+ y 2 m2+ y 3 m3 +… y n mn ) squirrel while moving upward (32 times).
C m− y = (
∑ mi (m 1 +m 2+ m 3 …+ mn ) 6.A new conveyor system at the local packaging plan will utilize a
along y−axis ¿ motor-powered mechanical arm to exert an average force of 890 N to
push large crates 12 meters in 22 seconds. Determine the power
C m−z =
∑ z i mi = ( z1 m1+ z 2 m2+ z 3 m3+ … z n mn) (
output required of such a motor.
∑ mi (m 1 +m 2+ m3 …+ mn )
7.The ski slopes at Bluebird Mountain make use of tow ropes to
along x−axis ¿ transport snowboarders and skiers to the summit of the hill. One of
Ticker tape diagram is another representation of the system’s the tow ropes is powered by a 22-kW motor which pulls skiers along
motion. A tick (dot) is placed on the tape at a fixed time interval as an icy incline of 14° at a constant speed. Suppose that 18 skiers with
it is stretch along the motion of the system. If distance between each an average mass of 48 kg hold onto the rope and suppose that the
tick is long, the velocity is fast. When distance between each tick is motor operates at full power.
short, the velocity is slow. If the distance is increasing, the system is
accelerating. If the distance is decreasing, the system is decelerating a. Determine the cumulative weight of all these skiers.
13 | P a g
e
b. Determine the force required to pull this amount of weight up a "Invariance of Physical Laws – University Physics Volume 3."
14° incline at a constant speed. https://opentextbc.ca/universityphysicsv3openstax/chapter/invarianc
e-of-physical-laws/.
c. Determine the speed at which the skiers will ascend the hill.

Activity 4 "RELATIVE MOTION ANALYSIS: VELOCITY." 07 May. 2016,


http://athena.ecs.csus.edu/~grandajj/e110/Lecture%20Notes%20for
Using the given Venn diagram below, differentiate center mass %20Section%2016-5-7.pdf
from geometric center and below the diagram is the box where
you will cite example in each. "Inertial frame of reference - Wikipedia."
https://en.wikipedia.org/wiki/Inertial_frame_of_reference.
Activity 4
"
Consider a collision of a tennis ball with a wall. Depending on the
I
physical properties of the ball and wall, the speed at which the
ball rebounds from the wall upon colliding with it will vary. The n
diagrams below depict the changes in velocity of the same ball. e
For each representation (vector diagram, velocity-time graph, rt
and ticker tape pattern), indicate which case (A or B) has the i
greatest change in velocity, greatest acceleration, greatest a
momentum change, and greatest impulse. Support each answer. l
I.

and Non-inertial Frame of Reference - QS Study."


https://qsstudy.com/physics/inertial-and-non-inertial-frame-of-
reference.

"Inertial and Non-inertial Frame of Reference - QS Study."


https://qsstudy.com/physics/inertial-and-non-inertial-frame-of-
reference.
REFERENCES
"Newton's First Law - Georgia State University."
"The Language of Physics: Physical Quantities and Units retrieved http://hyperphysics.phy-astr.gsu.edu/hbase/Newt.html.
from https://www.texasgateway.org/resource/13-language-physics-
physical-quantities-and-units "Newton’s Laws of Motion | Glenn Research Center | NASA." 25
May. 2021, https://www1.grc.nasa.gov/beginners-guide-to-
"Measurement Uncertainty | Boundless Chemistry." aeronautics/newtons-laws-of-motion/.
https://courses.lumenlearning.com/boundless-
chemistry/chapter/measurement-uncertainty/. "What Is Acceleration - Formula, Unit, Examples, Types, FAQs."
https://byjus.com/physics/acceleration/.
"Practices of Science: Precision vs. Accuracy | manoa ...."
https://manoa.hawaii.edu/exploringourfluidearth/physical/world- "Acceleration | Boundless Physics."
ocean/map-distortion/practices-science-precision-vs-accuracy https://courses.lumenlearning.com/boundless-
physics/chapter/acceleration/.
"What quantities are vectors." https://findanyanswer.com/what-
quantities-are-vectors. "Newton’s Second Law of Motion: Concept of a System | Physics."
https://courses.lumenlearning.com/physics/chapter/4-3-newtons-
"Scalars and Vectors - Physics Classroom." second-law-of-motion-concept-of-a-system/.
https://www.physicsclassroom.com/class/1DKin/Lesson-1/Scalars-
and-Vectors. "2.1 Relative Motion, Distance, and Displacement - Physics ...."
https://openstax.org/books/physics/pages/2-1-relative-motion-
"Acceleration Formula With Solved Examples." distance-and-displacement.
https://byjus.com/acceleration-formula/.
"NCERT Solutions for Class 11 Physics Chapter 3 Motion in a ...."
"Acceleration Formula With Solved Examples." https://byjus.com/ncert-solutions-class-11-physics/chapter-3-
https://byjus.com/acceleration-formula/. motion-in-a-straight-line/.
"kinematics - What does the magnitude of the acceleration ...." "One-Dimensional Kinematics: Motion Along a Straight Line." 23
https://physics.stackexchange.com/questions/98420/what-does-the- Jul. 2019, https://www.thoughtco.com/one-dimensional-kinematics-
magnitude-of-the-acceleration-mean. motion-straight-line-2698879.
"Relative Velocity: Definition, Concepts, Videos and Solved ...." "Invariance of Physical Laws - Physics LibreTexts." 11 Aug. 2021,
https://www.toppr.com/guides/physics/motion-in-a-straight- https://phys.libretexts.org/Courses/Muhlenberg_College/MC
line/relative-velocity/. %3A_Physics_121_-
_General_Physics_I/05%3A__Relativity/5.02%3A_Invariance_of_
"5.2: Invariance of Physical Laws - Physics LibreTexts." 11 Aug.
Physical_Laws.
2021, https://phys.libretexts.org/Courses/Muhlenberg_College/MC
%3A_Physics_121_- "Relative Velocity: Definition, Concepts, Videos and Solved ...."
_General_Physics_I/05%3A__Relativity/5.02%3A_Invariance_of_ https://www.toppr.com/guides/physics/motion-in-a-straight-
Physical_Laws. line/relative-velocity/.
14 | P a g
e
"Relativistic Addition of Velocities | Physics."
https://courses.lumenlearning.com/physics/chapter/28-4-relativistic-
addition-of-velocities/.

"Calculating the Amount of Work Done by Forces."


https://www.physicsclassroom.com/class/energy/Lesson-
1/Calculating-the-Amount-of-Work-Done-by-Forces.

"physics hw 6.docx - 1 a Work When a force acts upon an ...."


https://www.coursehero.com/file/79889361/physics-hw-6docx/.

"Work, Energy, Power, & Simple Machines - Nadirah's Physics ...."


https://sites.google.com/a/penncharter.com/nadirahs-
world/Home/unit-5--phat-physics/work--energy----power.

"Definition and Mathematics of Work - Physics Classroom."


https://www.physicsclassroom.com/Class/energy/u5l1a.html

"(PDF) Introduction to Fluid Mechanics | Masilakhe


Mgaguli ...."https://www.academia.edu/23546727/Introduction_to_
Fluid_Mechanics.

"AP Physics 1 by Edvantage Science - Issuu." 25 Aug. 2017,


https://issuu.com/edvantageinteractive/docs/ap_physics_1.

"Qrg_ee.pdf [d49o2pzwr149] - idoc.pub."


https://idoc.pub/documents/qrgeepdf-d49o2pzwr149.

15 | P a g
e
Republic of the Philippines Torque (τ ) – Torque is a force applied perpendicular to the radius
Department of Education
Region III – Central Luzon of circular motion (τ =F⊥ r =F ×rsin ( θ )) where θ is angle
SCHOOL DIVISION OF CITY OF BALANGA between Force applied and radius. This means that the farther the
BATAAN NATIONAL HIGH SCHOOL – SENIOR HIGH SCHOOL
City of Balanga, Bataan applied force from the axis of rotation, the greater the torque. The
same reason why mechanics uses extension of their wrench. Like in
linear motion, torque also results in change in angular velocity. This
NAME:_____________________Year &Section:_________ change in angular velocity is describe as angular acceleration ( α )
ACTIVITY SHEETS in GENERAL PHYSICS 1 and calculated by τ net=αI .
Quarter 2 - Week 1, Day 1- 4 (1. Calculate the moment of
inertia about a given axis of single-object and multiple-object
systems. 2. Calculate magnitude and direction of torque using
the definition of torque as a cross product. 3. Describe
rotational quantities using vectors. 4. Determine whether a
system is in static equilibrium or not. 5. Apply the rotational
kinematic relations for systems with constant angular
accelerations. 6. Determine angular momentum of different
systems. 7. Apply the torque-angular momentum relation. 8.
Solve static equilibrium problems in contexts but not limited to
see-saws, cable-hinge-strut-system, leaning ladders, and
weighing a heavy suitcase using a small bathroom scale)

Angular Velocity (ω ) – It
answers the question “how
fast the system rotates?” or
“how many rotations does
it make per unit of time?”
Angular velocity is
somehow similar to linear
velocity as linear velocity
is the distance traveled per
d
unit of time v=
t
however, it has a direction. The direction of a circular motion is
always changing thus, angular velocity is used instead. Angular
velocity also has directions but unlike linear velocity, it has
counterclockwise and clockwise direction. Counterclockwise
rotation is taken as positive angular velocity while clockwise is
negative. The distance traveled in a circular motion is part of the
circumference or the circumference itself thus, angular velocity is

ω= =2 πf where T is period (time it takes to complete 1
T
cycle
cycle) and f is frequency (how often, or Hertz Hz). Angular
s
rad
velocity is often expressed in (2 π rad =360° =1rev ¿
s

Angular Acceleration (α ) – It is the rate of change of angular


Δω
velocity per unit of time α =
Δt

Average Angular Velocity – It is the change in the angular


coordinate θ, expressed in radians, divided by the change in time.
The angular velocity is a vector that points in the direction of the
θ 2−θ1
axis of rotation. ω ave = (in rad/s)
t 2−t 1

Moment of Inertia ( I ) – It is the summation of all individual


masses of a system multiply by its radius squared ( I = ∑ m r 2).
Moment of inertia depends on some factors such as mass
distribution, radius from axis and axis of rotation. Here are some
example of shapes and its corresponding moment of inertia.

16 | P a g
e
Angular Momentum ( L) – is another property of rotating system. ¿ 57.24 x 10−3 kg ∙m 2
Similar to linear momentum, angular momentum is also a product
of mass and velocity ( P=mv ) only mass is angular inertia ( I ) and (b)
velocity is angular velocity (ω ) thus, L=Iω. τ 200 N ∙ m N
τ =αI ∴ α= = =3.494 x 103
Kinetic Rotational Energy – It is the kinetic energy due to the
I 57.24 x 10 kg ∙ m
−3 2
kg ∙ m
rotation of an object and is part of its total kinetic energy. ¿)
m
Static Equilibrium - If an object is at rest and is in a state of
kg ∙
s2 rad
equilibrium, then we would say that the object is at "static ¿ 3.494 x 10 3 =3.494 x 103
equilibrium." For rigid object, there are two conditions to be met kg ∙m s
first before saying it is in static equilibrium. First, its net force
3. The angular position of a 0.30-m-diameter flywheel is given by
should be equal to zero ∑ F=0. Forces should be equal and
opposite direction. The system’s weight, as an example, is a rad 3
downward force. Ground reaction should be equal to weight for net
force to be zero. 1st condition of equilibrium is met. Second
(
θ= 1.75
s3 )
t . (a) Find the average angular velocity, in

condition for equilibrium deals with torque. Net torque of the rad/s and in rev/min, over the time interval from t 1=1.5 s to
system should be zero ∑ τ=0 . Below is an example of a system t 2=4.0 s. (b) Find the angular displacement over that time
in a static equilibrium. Both net force and net torque were equal to interval.
zero
Solution:

Sample Problems:

1. The frequency of a spinning wheel is 30 Hz. (a) If the diameter


of the wheel is 50 cm, what is the angular speed (magnitude
without direction) of the wheel in radian per second? (b) What
is the period?

Solution:

rev
(a) f =30 Hz=30
s

2 π rad 30 rev rad


ω=2 πf = ∙ =188.496
rev s s

1 1
(b) T= = =33.333 ms
f 30 Hz

2. A machine part (as shown below) consists of three disks (with


radii r =0.04 m) linked by lightweight struts. (a) What is this
body’s moment of inertia about an axis through the center of
disk A, perpendicular to the plane of the diagram? (b) If a
200 N ∙m torque is applied, what is the angular acceleration of
the system? (same axis of rotation)

Solution:

1
(a) I =∑ m r 2=m B R B2+ mC RC 2 + m A r A2
2

1
¿ 0.10 kg ( 0.5 m )2 +0.20 kg ( 0.4 m )2 + ( 0.30 kg )( 0.04 m )2
2

17 | P a g
e
rad 3 rad 1. The angular position of a 0.36-m-diameter flywheel is
(a)
(
θ1= 1.75
s3 ) ( )
t 1 = 1.75 3 ( 1.5 s )3=5.906 rad
s rad 3
rad rad
given by θ= 2.0( s3 )
t . (a) Find the average angular

( s ) ( )
θ2= 1.75 3 t 23= 1.75 3 ( 4.0 s )3 =112 rad
s velocity, in rad/s and in rev/min, over the time interval
θ 2−θ1 112 rad−5.906 rad rad from t 1=2.0 s to t 2=5.0 s . (b) Fine the angular
ω ave = = =47.153 displacement of over that time interval. (c) Find the
t 2−t 1 4.0 s−1.75 s s
(b) instantaneous angular velocities at t 1=2.0 s and
rad rev 60 s rev t 2=5.0 s
47.153 × × =450.278 =θ 2−θ1=112 rad −5.906 rad=106.094 rad
s 2 π rad 1 min min Activity 2.B: Direction: Examine each figure below and
determine whether its 1st and 2nd condition of equilibrium
were satisfied or not. Explain your answer on each condition
(why satisfied or not) and tell the direction of motion (if system
Activity 1: Direction: Solve for the unknown quantities and will move).
simplify your answer to the nearest thousandths (3 decimal
places).

1. A machine part (as shown on the following figure)


consists of three disks linked by lightweight struts. (a)
What is this body’s moment of inertia about an axis
through the center of disk A, perpendicular to the plane
of the diagram? (b) What is its moment of inertia about
an axis through the centers of disks B and C?

2.
To

loosen a pipe fitting, a weekend plumber slips a piece of


scrap pipe (a “cheater”) over his wrench handle. He
stands on the end of the cheater, applying his full 900-N
weight at a point 0.80 m from the center of the fitting (as
shown below). The wrench handle and cheater make an
angle of with the horizontal. Find the magnitude and
direction of the torque he applies about the center of
the fitting.

Activity 2.A: Direction: Solve for the unknown quantities and


simplify your answer to the nearest thousandths (3 decimal
places).

18 | P a g
e
Activity 3: Direction: Solve for the unknown quantities and F=ma ) and his law of gravitation states what is mentioned above
simplify your answer to the nearest thousandths (3 decimal m 1 m2
places). ( F=G ), derivation dure to Earth’s gravity become
r2
1. We wrap a light, nonstretching cable around a solid possible. Both forces mentioned were equal if both forces are due to
gravity. Derivation is as follow:
cylinder of mass 50 kg and diameter 0.120 m, which
rotates in frictionless bearings about a stationary m1 m 2
horizontal axis (as shown below). We pull the free end of ma=G (Notice that masses m and m 1 are the same since
the cable with a constant 9.0-N force for a distance of
r2
they are both mass of the system. Acceleration a is also the
2.0 m; it turns the cylinder as it unwinds without
m2
slipping. Thecylinder is initially at rest. Find its final acceleration due to gravity therefore, g=G . Mass m2 now is
angular speed and the final speed of the cable r2
the mass of the Earth and radius r is Earth’ radius (since we are
dealing with the acceleration due to Earth’s gravity). Substituting
the know quantities,
k
−11 N ∙m 2 5.972 x 1024 kg N
g=6.674 x 10 2
× 2
=9.820 =9.820
kg ( 6.371 x 106 m ) kg

Gravitational Potential Energy - is energy an object possesses


because of its position in a gravitational field. As discussed before,
Energy is the ability to do work and the dot product of force and
displacement W =F ∙ dcos ( θ ) . Gravitational potential energy is a
2. The angular momentum of a rod changes from15 to conservative energy thus, force applied by the gravitational field is
always parallel to its displacement. With these, it is safe to assume
rad that angle θ is always zero (cos ( 0 )=1). It is also discussed that
35 kg ∙ m 2 ∙ in 4 seconds. What is the average
s force is a product of mass and acceleration and this acceleration is
torque acting on the rod? due to Earth’s gravity thus, W =U g =mgh. Distance d is equal to
height h since the motion is vertical.

3. A 10 kg disk of radius 3 meters is spinning at 15 Kepler’s Third Law - states that a planet’s orbital period is
rad/s. (a) What is the inertia of the disk? (b) proportional to the size of its orbit (its semi-major axis). Its
Calculate the angular momentum of the disk. relationship can be calculated as P2 α a3 where Pis period or time
it takes for one revolution and a is the semi-major axis. If period P
is in terms of Earth year (time it takes for Earth to completes its
Republic of the Philippines revolution around the Sun) and semi-major axis a is in terms of
Department of Education
Region III – Central Luzon astronomical unit AU (the distance between Earth and its semi
SCHOOL DIVISION OF CITY OF BALANGA major axis) then P2=a3. This means that the computed period P
BATAAN NATIONAL HIGH SCHOOL – SENIOR HIGH SCHOOL
City of Balanga, Bataan will be in relate to Earth’s period. Here is an example.

Relationship of Newton’s Law Gravitation and Centripetal


NAME:_____________________Year &Section:_________ Acceleration and Kepler’s Third Law of Planetary motion –
ACTIVITY SHEETS in GENERAL PHYSICS 1 Centripetal force is the force responsible for a circular motion to
Quarter 2 - Week 2, Day 1- 4 (1. Use Newton’s law of keep its path. This force is often due to tension of string that
gravitation to infer gravitational force, weight, and acceleration attached to the system. Planets and other massive objects have
due to gravity. 2. Discuss the physical significance of gravity instead of string to keep it in orbit. This means that
gravitational field. 3. Apply the concept of gravitational centripetal force and gravitational force of two massive object in
potential energy in physics problems. 4. Calculate quantities orbit are the same. The following shows the relationship of the two.
related to planetary or satellite motion. 5. For circular orbits,
relate Kepler’s third law of F C =FG (centripetal force is equal to gravitatio
planetary motion to Newton’s law of gravitation and centripetal
acceleration) m∙ v 2 m1 m2
=G ∙ 2
Newton’ Law of Gravitation – This law states that any particle of r r
matter in the universe attracts any other with a force varying
directly as the product of the masses and inversely as the square of Cancelation of m∧m 1 are made possible since they both represents
m 1 m2 the mass of a planet. m 2 represents Sun’s mass and radius r is the
the distance between them. Using mathematics; ( F=G )
r2 semi-major axis. Also remember that velocity discussed here is
where; G is gravitational constant and has a value of 2 π ∙r
angular velocity v= where P is the period and r is the
−11 N ∙ m
2 P
G=6.674 x 10 , m 1∧m2 are masses of object 1 and 2 radius or the semi-major axis.
kg 2 M Sun 4 π 2 ∙a 2 M Sun
2π ∙a 2
respectively and r is the distance between them.
( P ) =G
a

P2
=G ∙
a
Acceleration due to gravity – As Newton’s 2nd law of motion
states that a net force applied on a system produces acceleration ( Rearranging this term
19 | P a g
e
4π2 −11 m
3 ¿ 298.272 N
P=
2
G∙ M Sun
3
∙ a G=6.674 x 10
kg ∙ s2 ( ) a=G
m Mars
=6.674 x 10 −11 N ∙ m 2 6.42 x 1023 kg

(b)
Sample Problems: r Mars2 kg2 ( 3.390 x 106 m )2

m of6.42 x 1023 kg and a radius m


1. Planet Mars has mass kg ∙
r =3.390 x 106 m. (a) Find the gravitational force applied by N s2 m
¿ 3.728 =3.728 =3.728 2
Mars’ gravity on an astronaut with a mass of 80 kg . (b) What kg kg s
is the acceleration due to Mar’s gravity? (c) Mars is 215.53 (c)
million km away from the Sun, what is the force of gravity 23 3
between the two if Sun has a mass of 1.989 x 1030 kg?
m1 m2 N ∙ m2 ( 6.42 x 10 kg ) ( 1.989 x 10
F=G =6.674 x 10−11 ∙
r2 kg 2 2
( 215.53 x 109 m)
(a)
23
m1 m2 −11 N ∙ m2 ( 80 kg ) ( 6.42 x 10 kg )
F=G =6.674 x 10 ∙
r2 kg 2 2
( 3.390 x 106 m)

2. An apple with a mass of 100 g hanging on a tree is 3.5 m high 1. A box with a mass of 2500g is lifted onto a shelf which is 1 m
from the ground. (a) What is the gravitational potential energy of above the ground. Calculate the gravitational potential energy
the apple? gained by the box.
𝑚
(a) 𝑈𝑔 = 𝑚𝑔ℎ = ሺ0.1 𝑘𝑔ሻቀ9.8 2ቁሺ3.5 𝑚ሻ= 𝟑.𝟒𝟑 𝑱
𝑠
2. A10,000-kilogram satellite is pulled toward Planet A, with a
radius of 65,000 kilometers and a mass of 6.7 x 1033. When the
3. Mar’s semi-major axis is 1.524 AU or 228 x 106 km . (a) How satellite is 2,000 kilometers from the planet’s surface, what is
long will it take for Mar’s to complete is revolution to Sun the satellite’s acceleration (providing that its engines are turned
compared to Earth? (b) How long will it take for Mar’s to complete off)?
is revolution in terms of hours?
(a) 𝑃2 = 𝑎3 ∴ 𝑃 = ξ 𝑎3 = ඥሺ1.524ሻ3 = 𝟏. 𝟖𝟖𝟏 Earth years Activity 3: Direction: The following question is referring to the
concept of Kepler’s third law of planetary motion in relation to
(b) There two ways to answer this question. First is to convert
Newton’swhat
law of gravitation and centripetal acceleration.
is derived from problem (a) to days and second, is to use Kepler’s for the unknown quantities and round-off your final answers in
third law of planetary motion. the nearest hundredths.
1. It takes Mars 686.98 days to orbit around the Sun. If the Earth-
24 ℎ𝑜𝑢𝑟𝑠 Sun distance is 1.5 x 1011 m, what is the mean distance between
1.881 𝐸𝑎𝑟𝑡ℎ 𝑦𝑒𝑎𝑟𝑠 × 𝟏𝟔.𝟒𝟖𝟗 𝒙 𝟏𝟎𝟑 𝒉𝒓
1 𝐸𝑎𝑟𝑡ℎ 𝑑𝑎𝑦 Venus and the Sun?

4𝜋 2 4𝜋 2
𝑃2 = ∙ 𝑎3 ∴ 𝑃 = ට ∙𝑎3
𝐺∙𝑀𝑆𝑢𝑛 𝐺∙𝑀𝑆𝑢𝑛 The moon takes 27 days to revolve around the Earth. Calculate the
mass of the Earth using the Moon-Earth mean distance 3.8 x 10 x
4𝜋 2 ∙𝑘𝑔∙𝑠 2
= ටሺ6. ∙ ሺ228 𝑥 109 𝑚ሻ3 108 m.
674 𝑥 10−11 𝑚 3ሻሺ1.989 𝑥 1030 𝑘𝑔ሻ

1 ℎ𝑟 Republic of the Philippines


= 59.371 𝑥 106 𝑠 × = 𝟏𝟔.𝟒𝟗𝟐 𝒙 𝟏𝟎𝟑 𝒉
360 𝑠 Department of Education
Region III – Central Luzon
Activities: SCHOOL DIVISION OF CITY OF BALANGA
BATAAN NATIONAL HIGH SCHOOL – SENIOR HIGH SCHOOL
Activity 1: Direction: The following question is referring to the City of Balanga, Bataan
concept of Newton’s Law of Gravitation on inferring
gravitational force, weight, and acceleration due to gravity and NAME:_____________________Year &Section:_________
Physical significance of gravitational field. Solve for the
unknown quantities and round-off your final answers in the ACTIVITY SHEETS in GENERAL PHYSICS 1
nearest hundredths. Quarter 2 - Week 3, Day 1- 4 (1. Relate the amplitude,
frequency, angular frequency, period, displacement, velocity,
1. A married couple is standing on a bus stop waiting for a ride. and acceleration of oscillating systems. 2. Recognize the
The wife and the husband have a mass of 65kg and 80 kg, necessary conditions for an object to undergo simple harmonic
respectively. Calculate the gravitational force that exist motion. 3. Calculate the period and the frequency of spring
between them if they are 50cm apart from each other. mass, simple pendulum, and physical pendulum)

2. Simple harmonic motion- Is a special type of periodic motion


Planet A and Planet B have a mass of 4 x 10 24 kg and 4.5 x 1025
where the restoring force on the moving object is directly
kg. The gravitational force between the two planets is 4.8 x
proportional to the object's displacement magnitude and acts
1020. How apart is Planet A from Planet B?
towards the object's equilibrium position. An object moves with
simple harmonic motion whenever its acceleration is proportional to
Activity 2: Direction: The following question is referring to the its displacement from some
application gravitational potential energy in physics problems equilibrium position and is
and calculation of quantities related to planetary or satellite oppositely directed. Look at
motion. Solve for the unknown quantities and round-off
the your
figure as an example.
final answers in the nearest hundredths.
20 | P a g
e
Assumed that it is an ideal spring (no friction). When the object The amplitude of this wave is the displacement of the object from
attached to the spring is removed from its equilibrium position, its equilibrium point. It is said earlier that its elastic potential energy
spring’s force tries to bring it back to its equilibrium position. This is completely transform into kinetic energy.
fore is call restoring force ( F=−kx ) where k is the spring
constant ( N /m ¿ and x is the distance from its equilibrium
position. Notice also that restoring force is negative. This means
that the direction of this force is always towards its equilibrium. (the velocity of an oscillating system is also its angular velocity and
Once the force that cause the object to move away from its the linear displacement x is also its radius r )
equilibrium position is remove, restoring force will pull or push the
object back to its equilibrium point. Remember that ideal spring is 1 k 1 k
used therefore, spring’s elastic potential energy will be completely
transformed to kinetic energy at the equilibrium point and no loss
has been done. This kinetic energy will cause the object to move
f=
√ 2
∙ =
(2 π ) m 2 π √ m
Period is another property of sim

harmonic motion and is reciprocal of frequency


again away from its equilibrium point and will store again an elastic
potential energy. This process will keep on going and is called 1 m
simple harmonic motion. If the motion of the object is graphed
against time, sinusoidal wave will be formed.
T = =2 π
f k √
Angular velocity or angular frequency ω of a spring mass system is
k
calculate by ω=
√ m
(units are in rad/s)

Pendulum – pendulum’s motion has similarities with simple

F 6.0 N N kg
(a) k= = =200 =200 2
x 0.030 m m s

k 200 kg/ s2
(b) ω=
√ √m
=
0.50 kg
=20 rad /s

1 k 1 200 kg /s 2

harmonic motion however, it is not an exact simple harmonic


f=
2π √ =

m 2 π 0.50 kg
=3.183 Hz

motion. Its restoring force is due to gravitational force and not 1 1


always directly towards to its equilibrium position. As shown on the T= = =314.169 ms
f 3.183 Hz
figure, an object is attached to a string and will move left and right
passing its equilibrium point. Gravitational force, which is also its 2. Find the frequency and period of a simple pendulum 1.0 m
restoring force, is always directed downward. Only a portion of long at a location where g=9.8 m/s 2
gravitational force were used to restore the system into its
equilibrium position F Res=−mg∙ sin(θ). It is also safe to use 1 g 1 9.8 m/ s2
angular displacement (θ ) instead of liner displacement. Period
L 1 g
T is
f=
2π √ =
L 2π √ 1.0
=498.234 x 10−3 Hz

calculated as T =2 π
√ g
and frequency as f =
2π √ L T =2 π
√ L
g
=2 π
√1.0 m
9.8 m/s 2
=2.007 s
Sample Problems:

1. A spring is mounted horizontally, with its left end fixed. A


spring balance attached to the free end and pulled toward Activities:
the right (Fig. a) indicates that the stretching force is
proportional to the displacement, and a force of 6.0 N Activity 1: Direction: The following question is referring to the
causes a displacement of 0.030 m. We replace the spring concept of oscillating systems. Solve for the unknown
balance with a 0.50-kg glider, pull it 0.020 m to the right quantities and round-off your final answers in the nearest
along a frictionless air track, and release it from rest (Fig. hundredths.
b). (a) Find the force constant k of the spring. (b) Find the
1.A horizontal spring (K=250 N/m) with a mass of 500g attached to
angular frequency ω, frequency ƒ, and period T of the
it is undergoing simple harmonic motion. Calculate the (a) period,
resulting oscillation.
(b) frequency, and (c) angular frequency of this oscillator.

2.A horizontal spring with a 0.5kg block attached to it by 50cm is


stretched using a force of 400 N. The block is released from rest
and undergoes simple harmonic motion. Calculate the value of (a)
amplitude, and (b) maximum acceleration.

21 | P a g
e
Activity 2: Direction: Solve for the unknown quantities and
simplify your answer to the nearest thousandths (3 decimal
places).

1.A spring, with a spring constant of k=200 N/m, is mounted


horizontally with its left end fixed. If a glider with a mass of
m=0.70 kg is fixed on the other end and the spring is stretch 0.020
m from its equilibrium position, (a)how much force does the spring
exert on the glider? (b)what is its frequency? (c)what is its period?

2.Given the figure below, calculate the pendulum’s (a) restoring


force at the initial position, (b) frequency, and (c) period.

22 | P a g
e
Republic of the Philippines Sinusoidal wave – is a form of wave which follows sinusoidal
Department of Education
Region III – Central Luzon
pattern. Its amplitude can be a function of both displacement x and
SCHOOL DIVISION OF CITY OF BALANGA time t .
BATAAN NATIONAL HIGH SCHOOL – SENIOR HIGH SCHOOL
City of Balanga, Bataan
y ( x , t )= Asin ( kx ∓ ωt ∓ ϕ ) where A is maximum amplitude, k

NAME:_____________________Year &Section:_________ is wave number (k = ∧λ is wavelength), x is the horizontal
λ
ACTIVITY SHEETS in GENERAL PHYSICS 1 displacement of the wave, ω is the angular velocity (ω=2 πf ¿, t
Quarter 2 - Week 4, Day 1- 4 (1. Differentiate underdamped, is time and ϕ is phase shift. Notice that ϕ is added to the function.
overdamped, and critically damped motion. 2. Define This phase shift corrects the initial position of the wave. Sin and
mechanical wave, longitudinal wave, transverse wave, periodic Cosine wave generate identical wave but different in phase. This
wave, and sinusoidal wave. 3. From a given sinusoidal wave means that either function could be used as long as its phase is
function infer the speed, wavelength, frequency, period, corrected. Note in mind that angle used here is in terms of radian.
direction, and wave number. 4. Apply the inverse-square
relation between the intensity of waves and the distance from Wave velocity - distance traversed by a periodic, or cyclic, motion
the source.) per unit time (in any direction). It is simply the velocity of wave as
it travels thru the medium. ( v=λf )
Damped Oscillation - means an oscillation that fades away with
time. Examples include a swinging pendulum, a weight on a spring,
Sample Problem:
etc. There are three types of damped oscillation. Such types were
underdamped, overdamped and critically damped.
1. Find the sinusoidal function of the wave in the figure given
above.
Overdamped - The condition in which damping of an oscillator
causes it to return to equilibrium without oscillating; oscillator
moves more slowly toward equilibrium than in the critically y ( x , t )= Asin ( kx ∓ ωt ∓ ϕ )
damped system.
2π 2π
Critically damped - The condition in which the damping of an k= = =1.571 m−1
λ 4m
oscillator causes it to return as quickly as possible to its equilibrium
position without oscillating back and forth about this position.
The direction of the motion is going to the right therefore, ω is
negative. This will be explained more later.
Underdamped Oscillation - The condition in which damping of an
oscillator causes it to return to equilibrium with the amplitude
gradually decreasing to zero; system returns to equilibrium faster m
0.5
but overshoots and crosses the equilibrium position one or more ω=−2 πf and v s
times. v=λf ∴ f = = =125 x 10−3 Hz
λ 4m

ω=−2 π ( 125 x 10−3 Hz )=−785.398 x 10−3 s−1

The figure above shows that the wave starts at its positive peak, but
the function used is a sine function. A sine function should start at 0
since sin ( 0 ) =0. This means that the wave has a phase shift ϕ . To
determine the value of ϕ , the direction of the phase shift must be
determine first (+ϕ=shifted ¿ the ¿−ϕ=shifted ¿the ¿). A
π
sine wave begins at zero and increase as it approaches (
2

Mechanical Wave – it is a wave that is not capable of transmitting


sin ( π2 )=1¿ . It will then begin to decrease as angle increases until

its energy through a vacuum. Mechanical waves require a medium 3π 3π


in order to transport their energy from one location to another. In
simple words, it is the disturbance of the medium which travels
it reaches its negative peak at
2
(sin
2 ( )
=−1). Observe the
wave above, it begins at its positive peak then began to decrease its
along it. There are two classifications of mechanical wave and such
amplitude until it reaches its negative peak. This means that the sine
classifications are transverse wave and longitudinal wave.
π −3 π
wave was shifted by an angle ϕ= or ϕ= .
Transverse wave – is a wave where the displacement of the 2 2
medium is perpendicular with the direction of the motion of the
wave. y ( x , t )= Asin ( kx−ωt −ϕ )
Longitudinal wave - is a wave where the displacement of the
π
medium is parallel with the direction of the motion of the wave.
(
¿ 3 msin 1.571 m−1 x−785.398 x 10−3 s−1 t+
2 )
Periodic wave – is a wave that follows repetitive pattern within a
given time (Period) 2. From the given sinusoidal wave, find (a) wave number, (b)
wavelength, (c) frequency, and (d) period.

2.1.
y ( x , t )= A sin ( kx−ωt ) =5 msin ( 2.094 x−18.850 s−1 t )
(a) k =2.094 m−1

23 | P a g
e
2π 2π 2π Activities:
(b) k= ∴ λ= = =3 m
λ k 2.094 m−1
Activity 1: Direction: Answer the following question. Choose
(c) ω=2 πf ∴ f =
ω 18.850 s−1 the letter of the best answer. Use CAPITAL letter only.
= =3 Hz
2π 2π
1 1 1. Which of the following motion oscillates but decrease
(d) T = = =333.333 ms exponentially in amplitude?
f 3 Hz
A. Under damped motion
B. Critically damped motion
2.2.
C. Over damped motion
y ( x , t )= A sin ( kx−ωt ) =2 msin ( 0.393 m−1 x−12.566 s−1 t ) D. None of these
(a) k =0.393 m−1
2π 2π 2π 2. Which of the following motion return to the equilibrium the
(b) k = ∴ λ= = =15.988 m fastest?
λ k 0.393 m−1
(c) ω=2 πf ∴ f =
ω 12.566 s−1 A. Under damped motion
= =2 Hz
2π 2π B. Critically damped motion
1 1 C. Over damped motion
(d) T = = =500 ms D. None of these
f 2 Hz
3. Which of the following motion return to the equilibrium the
P slowest?
Intensity – is defined as the power delivered per unit area ( I = ).
A A. Under damped motion
Sound wave, as an B. Critically damped motion
example, produced C. Over damped motion
from its source D. None of these
travels in all
direction forming a
spherical shape. The 4. What type of mechanical wave is being described by the
area in this case is the following sentence? The displacement of the medium is
area of a sphere perpendicular to the direction of travel of the wave.
which is A=4 π r 2 . A. Periodic wave
Notice also that area B. Longitudinal wave
increases by squared as radius or distance from the source increases. C. Transverse wave
P D. Sinusoidal wave
(I = )
4 π r2
5. What type of mechanical wave is being described by the
Intensity in decibel (dB) - unit for expressing the ratio between following sentence? The displacement of the medium is along to the
two physical quantities, usually amounts of acoustic or electric direction of travel of the wave.
power, or for measuring the relative loudness of sounds. This level A. Periodic wave
of loudness of sound is called sound intensity level
B. Longitudinal wave
I
β ( dB )=10 log ( )
Io
. I 0 is the lowest or threshold intensity of C. Transverse wave
D. Sinusoidal wave
sound a person with normal hearing can perceive at a frequency of
W 6. Which of the following best define mechanical wave?
1000 Hz and has a constant value of I 0=10−12 .
m2 A. It is the disturbance that travels through some material or
substance called the medium.
Sample Problem: B. It is the disturbance that travels through some material or
substance called the medium and is repetitive.
1. A listener is 10 m away from a sound source. If the sound C. It is the disturbance that travels through some material or
generated by the source has a power of 100 W, how much substance called the medium and is not repetitive.
W D. None of the above
intensity will the listener hear (a) in and (b) in decibel
m2
β ( dB )? 7. Which of the following best define periodic wave?
A. It is the disturbance that travels through some material or
P 100 W W substance called the medium.
(a) I= = =79.577 x 10−3 2 B .It is the disturbance that travels through some material or
A 4 π ( 10 m ) 2
m substance called the medium and is repetitive.
C. It is the disturbance that travels through some material or
(b)
substance called the medium and is not repetitive.
I 79.577 x 10−3 D.None of the above
β ( dB )=10 log
( )
Io
=10 log ( 10−12 )
=99.008 dB
Activity 2.A: Direction: From the given sinusoidal function 1. 3 msin ( π m−1 x −1.57 s−1 t )
below, find its (a) wave number, (b) wavelength, (c) frequency,
and (d) period. 2. 2 msin ( 6.283 m−1 x−1.57 s−1 t )
y ( x , t )= A sin ( kx−ωt )
24 | P a g
e
3. 0.3 m sin ( 12.566 m−1 x−376.991 s−1 t ) And according to this, the net displacement of any component on
the string for a given time is equal to the algebraic totality of the
4. 0.1 msin ( 4.189 m−1 x−62.832 s−1 t ) displacements caused due to each wave. Hence, this method of
adding up individual waveforms for the evaluation of net waveform
5. 5 msin ( 2.094 m−1 x−18.850 s−1 t ) is termed as the principle of superposition.

6. 2.3 msin ( 0.898 m−1 x−31.416 s−1 t ) Let us say two waves are travelling alone and the displacements of
any element of these two waves can be represented by y1(x, t) and
7. 1 msin ( 0.628 m−1 x−94.248 s−1 t ) y2(x, t). When these two waves overlap, the resultant displacement
can be given as y(x,t).
8. 4 msin ( 0.571 m−1 x −150.796 s−1 t )
Mathematically: y (x, t) = y1(x, t) + y2(x, t)
9. 2 msin ( 0.393 m−1 x−12.566 s−1 t )
As per the principle of superposition, we can add the overlapped
waves algebraically to produce a resultant wave. Let us say the
Activity 2.A: Direction: From the given quantities of wave,
wave functions of the moving waves are
formulate a sinusoidal wave function with respect to x and t.
1. A=2 m, f =12 hz , k =15 m−1 y1 = f1(x–vt),
2. A=0.2 m, f =10 hz , λ=0.5 m
y2 = f2(x–vt)
m
3. A=1m , v=0.25 , λ=2 m thus
s
4. A=3 m, k =628.319 m−1 , ω=94.248 s−1 yn = fn (x–vt)

5. A=0.25 m, T =2 s , λ=3.14 m then the wave function describing the disturbance in the medium
−1 can be described as
6. A=0.12 m, T =3 s ,k =4.189m
7. A=0.25 m, f =2 hz , λ=6.283 m y = f1(x – vt)+ f2(x – vt)+ …+ fn(x – vt) or

m y=∑ i=1 to n = fi (x−vt)


8. A=0.23 m, v =2 , ω=150.796 s−1
s
A variety of standing waves can occur
9. A=11 m, T =1.5 s , λ=1 m when a string is fixed at both ends and set
into vibration. The vibrations on the
10. A=2.4 m , k=1.047 m−1 , ω=87.966 s−1 string of a musical instrument, such as
the violin, usually consist of many
standing waves together at the same
Activity 3: Direction: Answer the following question. Show your time, each of which has a different
solution and round-off your answer to the nearest thousandths wavelength and frequency. So, the
(3 decimal place) sounds you hear from a stringed The vibrating strings of a
violin produce standing
1. A concert was held on an open field. You and your friends instrument, even those that sound like waves whose frequencies
a
were there at the time and were 20 m away from the speaker. single pitch, actually consist of depend on the string
multiple frequencies. lengths.
(a)How much intensity of sound would you here if the speaker
has a 15,000 W of power and is used at its full capacity?
Superposition of Waves – As discussed earlier, is the addition of
(b)According to National Institute on Deafness and Other waves when they interfere
Communication Disorders, long or repeated exposure to sound with one another. Unlike
at or above 85 decibels can cause hearing loss. With this matter, wave can be at the
information, at least how much more should you and you friend same place at the same
move from your current position to prevent any hearing time. The resultant wave
damages? will be the sum of the
waves that interfere, as
shown beside.
Republic of the Philippines
Department of Education
Region III – Central Luzon
A R= A A + A B
SCHOOLS DIVISION OF CITY OF BALANGA
BATAAN NATIONAL HIGH SCHOOL – SENIOR HIGH SCHOOL Type of Wave Interface
City of Balanga, Bataan

NAME:_____________________Year &Section:_________ Constructive interface - If two waves superimpose with each other
in the same phase, the amplitude of the resultant is equal to the sum
ACTIVITY SHEETS in GENERAL PHYSICS 1 of the amplitudes of individual waves. Pulses were used to
Quarter 2- Week 5 Day 1-4 {Describe qualitatively and represents a less complicated wave.
quantitatively the superposition of waves; Apply the condition
for standing waves on a string; Relate the frequency (source
dependent) and wavelength of sound with the motion of the
source and the listener; Relate density, specific gravity, mass,
and volume to each other.}

Principle of Superposition
Let us take the example of a string wave to define the principle of
superposition of wave that is based on the superposition theorem.
25 | P a g
e
cubic centimeter or 1 gram per millimeter. Density is measured in
the units kg/m3. The density of water at 4.0°C is 1000 kg/m3. The
specific gravity has no unit for measure because the numerator and
the denominator of the formula are the same so they cancel each
other out.

ρsubstance
SG=
ρwater

Where: SG- Specific Gravity; ρ substance- density of substance/object;


Destructive interface - If two waves superimpose with each other ρ water-density of water
in opposite phase, the amplitude of the resultant is equal to the
difference in amplitude of individual waves. Pulses were used to Activities:
represents a less complicated wave.
Activity 1.A: Direction: Answer the following question. Show
your solution and round-off your answer to the nearest
thousandths (3 decimal place) whenever necessary.
1. The string of a guitar has a length of 610 mm. What will
be the wavelength of the standing wave on that string
on its:
a. 1st harmonic
b. 2nd harmonic
c. 3rd harmonic
d. 4th harmonic
e. 25th harmonic?

Fundamental Frequency: 2. A wave is said to be a standing wave if


v v _______________.
f 1= =
λ1 2 L a. its wavelength is longer than the length of the string
Where: L=length of a vibrating string; λ=wavelength , f= b. its wavelength is shorter than the length of the string
frequency, v=speed of the wave
c. its wavelength is equal to the length of the string
Harmonic Series: its wavelength is twice the length of the string divided by a
a series of frequencies that includes the fundamental frequency
and integral multiples of the fundamental frequency.
whole number n
v
f n=n Activity 1.B: Direction: B. Answer the following question. Show
2L your solution and round-off your answer to the nearest
Where: fn= frequency of the nth harmonic. thousandths (3 decimal place) whenever necessary.
n= harmonic number (1,2, 3….) 1. A car with a loudspeaker just passed by. You remembered what
you learned in Physics that there are change in sound frequency
Doppler Effect:
if source or listener is moving. If you were standing beside the
road and the car is traveling with a velocity of 40 km/h towards
Relative motion between a source of sound and an observer
can create changes in the observed frequency. This frequency shift you. What frequency will you hear if the speaker on the car
is known as the Doppler effect. The frequencies heard by the generates a frequency of 60 Hz?
observer can be described by the following two equations:
1.
v sound
'
f =f ( v sound −v source )
The first equation applies when the source of sound
is approaching the observer
Activity 2: Direction: Using the figure below, answer the
2. following questions.
v sound
'
f =f ( v sound + v source )
The second equation applies when the source of sound is moving
away from the observer.
1.

where f´ represents the apparent frequency and f represents the


actual frequency.

Specific gravity refers to the ratio of the density of an object and


the reference material. Furthermore, the specific gravity can tell us
if the object will sink or float in reference material. Besides, the Two pulses with both amplitudes of +3 were about to meet at the
reference material is water that always has a density of 1 gram per center. (a) What type of wave interface occur when the pulses
26 | P a g
e
met? (b) What will be the total amplitude of the pulse produce
by the interference of the two pulses? (c) After the interference,
will both pulses move the same velocity as they were before?
2. Two pulses with amplitudes of +3 and -3 were about to meet at
the center. (a) What type of wave interface occur when the
pulses met? (b) What will be the total amplitude of the pulse
produce by the interference of the two pulses? (c) After the
interference, will both pulses move the same velocity as they
were before?

27 | P a g
e
Republic of the Philippines Applying Pascal’s principle and the definition of pressure gives the
Department of Education
following equation:
Region III – Central Luzon
SCHOOLS DIVISION OF CITY OF BALANGA
BATAAN NATIONAL HIGH SCHOOL – SENIOR HIGH SCHOOL F1 F2
City of Balanga, Bataan Pinc= =
A1 A 2
NAME:_____________________Year &Section:_________ Rearranging this equation to solve for F2 produces the following:
ACTIVITY SHEETS in GENERAL PHYSICS 1
Quarter 2- Week 6 Day 1-4 (Relate pressure to area and force; A2
Relate pressure to fluid density and depth; Apply Pascal’s F 2= F
A1 1
principle in analyzing fluids in various systems; Apply the
concept of buoyancy and Archimedes’ principle.)
This second equation shows that the output force, F2, is larger than
the input force, F1, by a factor equal to the ratio of the areas of the
We begin our study with fluid statics, the study of fluids at
two pistons. However, the input force must be applied over a longer
rest in equilibrium situations. Like other equilibrium situations, it is
distance; the work required to lift the truck is not reduced by the use
based on Newton’s first and third laws. We will explore the key
of a hydraulic lift.
concepts of density, pressure, and buoyancy.
Buoyancy
Fluid dynamics, the study of fluids in motion, is much Have you ever wondered why things fee
more complex; indeed, it is one of the most complex branches of than they do in air? The reason is that a fluid exerts an upward force
mechanics. on objects that are partially or completely submerged in it. This
upward force is called a buoyant force. If you have ever rested on
Pressure is a measure of how much force is an
applied over a in a swimming pool, you have experienced a
air mattress
given area. It can be written as follows: buoyant force. The buoyant force kept you and the mattress afloat.

F Force Because the buoyant force acts in a direction opposite the


P= Pressure=
A Area force of gravity, the net force acting on an object submerged in a
fluid, such as water, is smaller than the object’s weight. Thus, the
A=2 πrh+2 π r 2 object appears to weigh less in water than it does in air. The weight
Area of a cylinder of an object immersed in a fluid is the object’s apparent weight. In
the case of a heavy object, such as a brick, its apparent weight is
The SI unit of pressure is the pascal (Pa), which is equal to less in water than its actual weight is in air, but it may still sink in
1N/m2. The pascal is a small unit of pressure. The pressure of the water because the buoyant force is not enough to keep it afloat.
atmosphere at sea level is about 1.01 × 105 Pa. This amount of air
pressure under normal conditions is the basis for another unit, the Archimedes’ principle: When a body is completely or
atmosphere (atm). For the purpose of calculating pressure, 105 Pa is partially immersed in a fluid, the fluid exerts an upward force on the
about the same as 1 atm. The absolute air pressure inside a typical body equal to the weight of the fluid displaced by the body.
automobile tire is about 3 × 105 Pa, or 3 atm. F B=F g ( displaced fluid )=m f g
magnitude of buoyant force = weight of fluid displaced
When you pump a bicycle tire, you apply a force on the
pump that in turn exerts a force on the air inside the tire. The air
responds by pushing not only against the pump but also against the
walls of the tire. As a result, the pressure increases by an equal
amount throughout the tire.
In general, if the pressure in a fluid is increased at any point in a
container (such as at the valve of the tire), the pressure increases at
all points inside the container by exactly the same amount.

PASCAL’S PRINCIPLE
“Pressure applied to a fluid in a closed container is transmitted
equally to every point of the fluid and to the walls of the container.”

F B=F g ( object )=m o g


buoyant force = weight of floating object

Sample Problem:
A 15.0-kg solid gold statue is raised from the sea bottom. What is
the tension in the hoisting cable (assumed massless) when the statue
is (a) at rest and completely underwater and (b) at rest and
completely out of the water?

A hydraulic lift, such as the one shown in the figure, makes use of
Pascal’s principle. A small force F1 applied to a small piston of area
A1 causes a pressure increase in a fluid, such as oil. According to Solution:
Pascal’s principle, this increase in pressure, Pinc, is transmitted to a In both cases the statue is in equilibrium and experiences three
larger piston of area A2 and the fluid exerts a force F2 on this piston. forces: its weight, the cable tension, and a buoyant force equal in

28 | P a g
e
magnitude to the weight of the fluid displaced by the statue 5. Crawling in a frozen lake decreases the chance of breaking
(seawater in part (a), air in part (b)). The figure shows the the ice rather than walking on it. Which of the following explains
free-body diagram for the statue. Our target variables are the values the reason behind?
of the tension in seawater (Tsw) and in air (Tair). We are given the mass A. More body heat is transferred to the ice when crawling
mstatue, and we can calculate the buoyant force in seawater B. The body is heavier when walking than crawling
(Bsw) and in air (Bair) using Archimedes’ principle. C. Crawling put lesser pressure on the ice rather than walking
D. Walking put lesser pressure on the ice rather than crawling
(a) To find Bsw, we first find the statue’s volume V using
the density of gold 19.3 x 103 kg/m3. Activity 1.B: Direction: Answer the following question. Show
your complete solution and round-off your final answer in the
mstatue 15.0 kg nearest thousandths (3 decimals).
V= = 3
=7.77 x 104 m3
ρ gold 10 kg 1. How much pressure will be measure at the bottom of the water
19.3 x
m3 tank with a height of h=2 m and a cross sectional area of
A=785.398 mm2?
The buoyant force Bsw equals the weight of this same volume of 2. How much pressure will be measure at the middle of the water
seawater. tank with a height of h=2 m and a cross sectional area of
Bsw =wsw =msw g=ρ sw Vg
= (1.03 x 103 kg/m3) (7.77 x 10-4 m3) (9.80 m/s2)
A=785.398 mm2?
=7.84 N
The statue is at rest, so the net external force acting on it is zero.
∑ F y =B sw +T sw+ (−mstatue g )=0 3. If the diameter of the tank in the previous questions were
2
Tsw = mstatueg - Bsw = (15.0 kg) (9.80 m/s ) - 7.84 N increased twice but it height remain the same, what will happen
= 147 N - 7.84 N = 139 N to pressure (a) at the middle of the tank and (b) at the bottom of
the tank?
A spring scale attached to the upper end of the cable will indicate a
tension 7.84 N less than the statue’s actual weight mstatueg= 147N.
(b) The density of air is about 1.2 kg/m 3, so the buoyant force of air
on the statue is Activity 2: Direction: Solve for the unknown quantities and
simplify your answer to the nearest thousandths (3 decimal
kg
Bair =ρair Vg= 1.2
( 3
( 7.77 x 10−4 m3 ) 9.902 m
) ( ) places).
m s
-3
= 9.1 x 10 N 1. A downward force of 2500 N is applied to the smaller piston
This is negligible compared to the statue’s actual weight with a diameter of 0.5 cm in the hydraulic lift system shown
mstatueg=147 N. So within the precision of our data, the tension in below. (a) what is the upward force exerted by the larger piston
the cable with the statue in air is Tair=mstatueg=147 N. with a diameter of 1.5 cm? (b) If force applied on the smaller
piston pushes it down by 1m, how high will the larger piston
move?
Activities:
Activity 1.A: Direction: Answer the following question. Choose
the letter of the best answer and use CAPITAL letters only.
1. What is the density of water at room temperature?
g kg
A. 1000 B. 1
ml m3
g g
C. 1000 D. 1
cc cc
2. Two animals were placed on both end of a hydraulic lift system
2. What is the specific gravity of aluminum if its density is as show below. A mouse with a mass of m=9 g is placed on a
kg smaller piston with a diameter of 10 cm. If an elephant with a
ρal =2700 ? mass of 4 000 kg is placed on the larger piston, how wide
m3
should the diameter of the larger piston be so that pressures at
A. 2.7 x 10−12 B. 2.7 x 10 6
both ends are equal?
C. 2.7 x 10 12 D. 2.7
3. What is the specific gravity of gold if its density is
kg
ρau=19 300 ?
m3
A. 19.3 x 10−12 B. 19.3 x 106
C. 19.3 x 1012 D. 19.3
3. How much volume does a 200 kg of water(H20) occupies?
A. 0.200 m3 B. 0.020 m 3
C. 0.002 m 3 D. 2.000 m 3
4. A cube with a weight of 50 kg is sitting above the table. If
the cube has a side length of 2m, how much pressure does the cube
exert on the table? Activity 3: Direction: Solve for the unknown quantities and
A. 12.5 Pa B. 25 Pa simplify your answer to the nearest thousandths (3 decimal
C. 122.5 Pa C. 245 Pa places).
1. A gold with a mass of 20 kg is fully submerge on the sea water.
If a rope is used to hoist the gold up and the mass of the hoist is
29 | P a g
e
neglected, (a) what is the tension on the hoisting cable when
gold is completely under water? (b) what is the tension on the
hoisting cable when gold is completely out of water? (
kg kg kg
ρ sw=1023 3
, ρair =1.28 3 ∧ρ gold =19300 3 ).
m m m

2. A buoy has a mass of 950 kg and a diameter of 0.900 m,


calculate the additional distance the buoy will sink if a man
with a mass of 75 kg decided to stand on top of it. Figure 1
3. A system is placed on top of the water. What will happen to the We can discover an important property of thermal equilibrium by
system if (a) the density of the system is much greater than the considering three systems, A, B, and C, that initially are not in
water? (b) the density of the system is much lesser than the thermal equilibrium (Fig. 1). We surround them with an ideal
water? (c) the density of the system is equal to the water? insulating box so that they cannot interact with anything except
each other. We separate systems A and B with an ideal insulating
wall (the green slab in Figure A), but we let system C interact with
both systems A and B. This interaction is shown in the figure by a
Republic of the Philippines
yellow slab representing a thermal conductor, a material that
Department of Education permits thermal interactions through it. We wait until thermal
Region III – Central Luzon equilibrium is attained; then A and B are each in thermal
SCHOOLS DIVISION OF CITY OF BALANGA equilibrium with C. But are they in thermal equilibrium with each
BATAAN NATIONAL HIGH SCHOOL – SENIOR HIGH SCHOOL
City of Balanga, Bataan
other?

To find out, we separate system C from systems A and B with an


NAME:_____________________Year &Section:_________ ideal insulating wall (Figure B), and then we replace the insulating
wall between A and B with a conducting wall that lets A and B
ACTIVITY SHEETS in GENERAL PHYSICS 1 interact. What happens? Experiment
Quarter 2- Week 7 Day 1-4 (Apply Bernoulli’s principle and shows that nothing happens; there are no additional changes to A or
continuity equation, whenever appropriate, to infer B. We conclude:
relations involving pressure, elevation, speed, and flux; Explain
the connection between the Zeroth Law of Thermodynamics, “If C is initially in thermal equilibrium with both A and B, then A
temperature, thermal equilibrium, and temperature scales; and B are also in thermal equilibrium with each other. This result
Convert temperatures and temperature differences in the is called the zeroth law of thermodynamics.”
following scales: Fahrenheit, Celsius, Kelvin; Define coefficient
of thermal expansion and coefficient of volume expansion; Now suppose system C is a thermometer. In Fig. A, the
Calculate volume or length changes of solids due to changes in thermometer C is in contact with both A and B. In thermal
temperature; and Solve problems involving temperature, equilibrium, when the thermometer reading reaches a stable value,
thermal expansion, heat capacity, heat transfer, and the thermometer measures the temperature of both A and B; hence
thermal equilibrium in contexts such as, but not A and B both have the same temperature. Experiment shows that
limited to, the design of bridges and train rails thermal equilibrium isn’t affected by adding or removing insulators,
using steel, relative severity of steam burns and so the reading of thermometer C wouldn’t
water burns, thermal insulation, sizes of stars, change if it were in contact only with A or only with B. We
and surface temperatures of planets.) conclude:
Bernoulli’s Principle “Two systems are in thermal equilibrium if and only if they have
the speed of fluid flow can vary along the paths of the fluid. The the same temperature.”
pressure can also vary; it depends on height as in the static situation,
and it also depends on the speed of flow. We can derive an This is what makes a thermometer useful; a thermometer actually
important relationship called Bernoulli’s equation that relates the measures its own temperature, but when a thermometer is in
pressure, flow speed, and height for flow of an ideal, thermal equilibrium with another body, the temperatures must be
incompressible fluid. Bernoulli’s equation is an essential tool in equal. When the temperatures of two systems are different, they
analyzing plumbing systems, hydroelectric generating stations, and cannot be in thermal equilibrium.
the flight of airplanes.
First Law of Thermodynamics
This is Bernoulli’s equation. Also known as Law of Conservation of Energy, states that energy
can neither be created nor destroyed; energy can only be transferred
1 1 or changed from one form to another. For example, turning on a
p1 + ρg y 1+ ρ v 1 =p 2+ ρg y 2 + ρ v 2
2 2
2 2 light would seem to produce energy; however, it is electrical energy
that is converted.
It states that the work done on a unit volume of fluid by the
surrounding fluid is equal to the sum of the changes in kinetic and Second Law of Thermodynamics
potential energies per unit volume that occur during the flow. Says that the entropy of any isolated system always increases.
Isolated systems spontaneously evolve towards thermal equilibrium
The subscripts 1 and 2 refer to any two points along the flow tube, —the state of maximum entropy of the system. More simply put:
so we can also write the entropy of the universe (the ultimate isolated system) only
increases and never decreases.
1
ρ+ ρgy + ρ v 2=constant Third Law of Thermodynamics
2 States that the entropy of a system approaches a constant value as
the temperature approaches absolute zero. The entropy of a system
Note that when the fluid is not moving (so v1=v2=0)
at absolute zero is typically zero, and in all cases is determined only
by the number of different ground states it has. Specifically, the
The Zeroth Law of Thermodynamics
entropy of a pure crystalline substance (perfect order) at absolute
30 | P a g
e
zero temperature is zero. This statement holds true if the perfect Where ΔL is the change in length L, ΔT is the change in
crystal has only one state with minimum energy. temperature, and α is the coefficient of linear expansion, which
varies slightly with temperature.
Temperature
The temperature scales most widely used today are the Fahrenheit, Thermal Expansion in Two Dimensions
Celsius, and Kelvin scales. The Fahrenheit scale is commonly used For small temperature changes, the change in area ΔA is given by
in the United States. The Celsius scale is used in countries that have ΔA = 2αAΔT
adopted the metric system and by the scientific community
worldwide. Celsius and Fahrenheit temperature measurements can Where ΔA is the change in area A, ΔT is the change in temperature,
be converted to each other using this equation. and α is the coefficient of linear expansion, which varies slightly
with temperature.

Fahrenheit to Celsius Temperature Conversion Thermal Expansion in Three Dimensions


The change in volume ΔV is very nearly ΔV = 3αVΔT. This
9 equation is usually written as
T F = T C + 32.0
5
ΔV = βVΔT
9
Fahrenheit Temperature= ( x Celsius Temperature) + 32.0
5 Where β is the coefficient of volume expansion and β ≈ 3α.
Celsius to Fahrenheit Temperature Conversion Molar Heat Capacity
Sometimes it’s more convenient to describe a quantity of substance
T C =( T F −32 ) ( 59 ) in terms of the number of moles n rather than the mass m of
material. Recall from your study of chemistry that a mole of any
pure substance always contains the same number of molecules. The
Celsius Temperature= (Fahrenheit Temperature-32)
( 59 ) molar mass of any substance, denoted by M, is the mass per mole.
(The quantity M is sometimes called molecular weight, but molar
mass is preferable; the quantity depends on the mass of a molecule,
Celsius to Kelvin Temperature Conversion not its weight.) For example, the molar mass of water is 18.0
g/mol=
T K =T C +273.15 18.0 x 10-3kg/mol; 1 mole of water has a mass of 18.0 g = 0.0180
Kelvin Temperature=Celsius Temperature + 273.15 kg. The total mass m of material is equal to the mass per mole
M times the number of moles n:

Matter expands as its temperature increases m = nM


Increasing the temperature of a gas at constant pressure causes the
volume of the gas to increase. This increase occurs not only for The product Mc is called the molar heat capacity (or molar specific
gases but also for liquids and solids. In general, if the temperature heat) and is denoted by C (capitalized).
of a substance increases, so does its volume. This phenomenon is
known as thermal expansion. Q=nC ∆ T
You may have noticed that the concrete roadway segments of a We can express the molar heat capacity C (heat per mole per
bridge are separated by gaps. This is necessary because concrete temperature change) in terms of the specific heat c (heat per mass
expands with increasing temperature. Without these gaps, thermal per temperature change) and the molar mass M (mass per mole):
expansion would cause the segments to push against each other, and 1 dQ
they would eventually buckle and break apart. Different substances C= =MC (molar heat capacity)
undergo different amounts of expansion for a given temperature n dT
change. The thermal expansion characteristics of a material are
indicated by a quantity called the coefficient of volume expansion.
Gases have the largest values for this coefficient. Liquids have Activities:
much smaller values.
Activity 1: Direction: Solve for the unknown quantities and
In general, the volume of a liquid tends to decrease with decreasing simplify your answer to the nearest thousandths (3 decimal
temperature. But, the volume of water increases with decreasing places).
temperature in the range between 0°C and 4°C. Also, as the water
1. The water flows horizontally through a cylindrical tube with
freezes, it forms a crystal that has more empty space between the
molecules than does liquid water. This explains why ice floats in decreasing cross-sectional area as shown below. Before the
liquid water. It also explains why a pond freezes from the top down water enters point A, it has a pressure of 300 000 Pa and a
instead of from the bottom up. If this did not happen, fish would velocity of 5 m/s. Find the pressure and water velocity at point
likely not survive in freezing temperatures. Solids typically have the (a)A with a diameter of 30 cm, (b)B with a diameter of 20 cm,
smallest coefficient of volume expansion values. For this reason, and (c)C with a diameter of 30 cm.
liquids in solid containers expand more than the container. This
property allows some liquids to be used to measure changes in
temperature.

Linear Thermal Expansion- Thermal Expansion in One


Dimension
The change in length ΔL is proportional to length L. The
dependence of thermal expansion on temperature, substance, and
length is summarized in the equation:

ΔL = αLΔT

31 | P a g
e
2. From the situation below, find the fluid’s velocity and pressure at C. both systems have high temperature
point (a) A, (b) B, and (c) C if the fluid is pure water. D. systems have the same temperature
2. According to Zeroth Law of Thermodynamics, which of the
following statement is true about system A and system C in
Activity 2: Direction: Answer the following question. Choose contact if system A is in thermal equilibrium with system B and
the letter of the best answer. Use CAPITAL letter only. system B is in thermal equilibrium with thermal C?
A. System A in equilibrium with system C thus, not heat flow.
1. When two systems do not exhibit heat flow, this means that B. System A is not in equilibrium with system C and heat will
________. flow from system A to B and vice-versa.
A. one of the systems has a higher temperature C. Heat will flow from system A to system C.
B. both systems have low temperature D. Heat will flow from system C to system A.
3. A glass of hot water is placed on top of a table. After some
time, the temperature of the water is in equilibrium of its
surrounding. Which of the following laws of thermodynamics
explain this scenario?
A. Zeroth Law of Thermodynamics
B. First Law of Thermodynamics
C. Second Law of Thermodynamics
D. Third Law of Thermodynamics
4. The measurement shown on temperature scales is the
temperature of ______________.
A. the system that is being measured
B. the surrounding environment
C. the temperature scale itself
D. the temperature of the other system
5. What is the direction of thermal flow between the two systems
in contact if only one of the systems is in absolute zero
temperature?
A. Heat will always flow towards the system with absolute zero
temperature
B. Heat will always flow away from the system with absolute zero
temperature
C. Heat will not flow in any direction
D. It’s hard to tell since the temperature of the other system is
unknown
6. We often hear the phrase “sarado mo yung pinto kasi lalabas
yung aircon (lamig ng aircon)”. Is this statement true? or false?
Explain your answer

Activity 3: Direction: Convert the following temperature scales


to the desired scale. Show your solutions on your answer sheets.

A. °C to °F
1. 10 °C ________
2. 30 °C ________
3. 40 °C ________

B. °F to °C
4. 45 °F ________
5. 70 °F ________
6. 80 °F ________

C. °F to K
7. 90 °F ________
8. 212 °F ________

9. During the 24-hour period, a steel rail varies in


temperature from 35 °F at night to 65 °F in the middle of the day.
Express this range of temperature in degrees Celsius.

10. The melting point of lead is 330 °C. What is the


corresponding temperature on the Fahrenheit scale?

Activity 2: Direction: Write A if the statement is correct and if


it is incorrect change the underlined work/s that make the
statement incorrect.

1.The liquids and gases only have cubical expansion


2.If a metal bar is heated significantly, its expansion will be in all
directions
32 | P a g
e
3.Linear thermal expansion is best observed in solids
4.The chemical composition of the solid affects its coefficient of
linear expansion.
5.The coefficient of volume expansion of liquid water is always the
same
6.Gas is more responsive to temperature changes
7.Mercury thermometers are impractical for very cold climates
because Mercury Has a freezing point that is not low enough
8.Thermoscope is the first instrument the measured temperature
9.Absolute zero is equal to 0 °C
10.Solid expands more as compare to fluids.

Activity 3: Direction: Solve for the unknown quantity. Show


your solution.

1.An iron pipe is 50 m long at room temperature, (32 °C). If this


pipe is to be used as a steam pipe, how much allowance must be
made for expansion, and what will be the new length of pipe after
steam has been flowing for a while?
2.A brass disk has a hole of 90 mm in diameter punched in its
center. Later, the disk at 25 °C is placed into boiling water for a
few minutes. What will be the new area of the hole?
3.A pyrex glass bulb is filled with 40 cm3 of mercury at 23 °C.
What volume will overflow if the system is heated uniformly to a
temperature of 60 °C?
4.A square copper plate 6 cm on a side at 23 °C is heated to 120 °C.
What is the increase in the area of the copper plate?
5.What is the increase in volume of 18 liters of ethyl alcohol when
the temperature is increase from 18 °C to 30 °C?
Activity 4: Direction: Solve for the unknown quantity. Show
your solution.

1.A steel tape measures the length of an aluminum rod as 60 cm


when both are at 8 °C. What will the tape read for the length of the
rod if both are at 38 °C?
2.A round brass plug has a diameter of 6.0 cm at 30 °C. To what
temperature must the plug be cooled if it is to fit snugly into an 8
cm hole?
3.A steel ring has an inside diameter of 6.0 cm at 23 °C. The ring is
to be slipped over a copper shaft that has a diameter of 6.005 cm at
23 °C. To what temperature must the ring be heated? If the ring
and the shaft are cooled uniformly, at what temperature will the ring
just slip off the shaft?
4.A handful of copper shot is heated to 100 °C and then dropped
into 160 g of water at 23 °C. The temperature of the mixture is 27
°C. What was the mass of the shot?
5.What quantity of heat is required to change the temperature of
200 g of lead from 20 to 100 °C?

33 | P a g
e
Republic of the Philippines
Department of Education
M kB T M k B T ρ kB T
Region III – Central Luzon
SCHOOLS DIVISION OF CITY OF BALANGA
BATAAN NATIONAL HIGH SCHOOL – SENIOR HIGH SCHOOL
P=
mV
= ( )
V m
=
m
City of Balanga, Bataan
Where: P=pressure (N/m2); V=volume (m3); N=number of gas
particles; kB=Boltzmann’s constant (1.38x10-23 J/K); T=
NAME:_____________________Year &Section:_________ temperature (K); m=mass of gas (Kg); n= moles; M= molar mass
ACTIVITY SHEETS in GENERAL PHYSICS 1 (Kg/mol)
Quarter 2- Week 8 Day 1-4 (Enumerate the properties of an
ideal gas; Solve problems involving ideal gas equations in
contexts such as, but not limited to, the design of metal A third way of writing the ideal gas law may be familiar to you
containers for compressed gases; Interpret PV diagrams of a from your study of chemistry:
thermodynamic process; Compute the work done by a gas using PV = nRT
dW=PdV; State the relationship between changes internal In this equation, n is the number of moles of gas (one mole is equal
energy, work done, and thermal energy supplied through the to 6.02 × 1023 particles). The quantity R is a number called the
First Law of Thermodynamics; Differentiate the following molar (universal) gas constant and has a value of 8.3 1 J/(mol•K).
thermodynamic processes and show them on a PV diagram:
isochoric, isobaric, isothermal, adiabatic, and cyclic; Calculate
the efficiency of a heat engine; Describe reversible and PV Diagram
irreversible processes; Explain how entropy is a measure of The PV diagram models the relationship between pressure (P) and
disorder; State the 2nd Law of Thermodynamics; Calculate volume (V) for an ideal gas. The processes plotted on PV diagrams
entropy changes for various processes e.g., isothermal process, only work for a closed system (in this case the ideal gas), so there is
free expansion, constant pressure process, etc.) no exchange of matter, but still an exchange of energy.

When the density of a gas is sufficiently low, the pressure, volume, To understand how pressure directly affects volume (and vice
and temperature of the gas tend to be related to one another in a versa)—imagine a sealed container, containing an ideal gas (the
fairly simple way. This relationship is a good approximation for the system), that has a moving piston. If a force is applied, the piston
behavior of many real gases over a wide range of temperatures and moves down, and the gas would compress—decreasing the volume
pressures. These observations have led scientists to develop the in the system and causing an increase in pressure. Moreover, if the
concept of an ideal gas. piston moves up, the volume of the system would increase,
decreasing the pressure of the system. Therefore, an increase in one
Volume, pressure, and temperature are the three variables that variable will cause the decrease in the other, and vice versa.
completely describe the macroscopic state of an ideal gas. One of However, if an increase (or decrease) in pressure and/or volume is
the most important equations in fluid mechanics relates these three desired, an external heat source (or a cooling source) from its
quantities to each other. surroundings must be added.

The Ideal Gas Law Ideal Gas Equation for Thermodynamic Processes
is an expression that relates the volume, pressure, and temperature
of a gas. This relationship can be written as follows:

PV =N k B T

Where: P=pressure (N/m2); V=volume (m3); N=number of gas


particles; kB=Boltzmann’s constant (1.38x10-23 J/K); T=temperature
(expressed in Kelvin)

The ideal gas law makes no mention of the composition of the gas.
The gas particles could be oxygen, carbon dioxide, or any other gas.
In this sense, the ideal gas law is universally applicable to all gases.

If a gas undergoes a change in volume, pressure, or temperature (or


any combination of these), the ideal gas law can be expressed in a
particularly useful form. If the number of particles in the gas is
constant, the initial and final states of the gas are related as follows:

N 1=N 2

P1V 1 P2V 2
=
T2 T2

Where: P=pressure; V=volume; T=temperature

According to the ideal gas law, when the temperature increases,


either the pressure or the volume—or both—must also increase.

Another alternative form of the ideal gas law indicates the law’s
dependence on mass density. Assuming each particle in the gas has
a mass m, the total mass of the gas is N × m = M. The ideal gas law
can then be written as follows:
M kB T
PV =N k B T =
m
34 | P a g
e
Four Main Ideal Gas Law processes modelled on PV Diagrams Recall from the first law of thermodynamics that the work done on
the environment by the engine is equal to the difference between the
energy transferred to and from the system as heat. For a heat
engine, the efficiency is the ratio of work done by the engine to the
energy added to the system as heat during one cycle.

Equation for the efficiency of Heat Engine

W net Q h −Q c Qc
eff = = =1−
Qh Qh Qh

net work done by engine


efficieny = engine as heat ¿
energy added ¿

energy added as heat−energy removed as heat


¿
energy added as heat

energy removed as heat


¿ 1−
energy added as heat
Notice that efficiency is a unitless quantity that can be calculated
using only the magnitudes for the energies added to and taken away
from the engine. This equation confirms that a heat engine has 100
percent efficiency (eff = 1) only if there is no energy transferred
away from the engine as heat (Qc = 0).

Second Law of Thermodynamics


Any thermodynamic processes proceed naturally in one direction
but not the opposite. For example, heat by itself always flows from
a hot body to a cooler body, never the reverse. We can also state the
second law in terms of the concept of entropy, a quantitative
measure of the degree of disorder or randomness of a system. The
WORK (expressed in Joule/J) idea of entropy helps explain why ink mixed with water never
Work done in volume changes spontaneously unmixes.

V2 Reversible Process
W =∫ pdV  system and surroundings can be restored to the initial state
V1
from the final state without producing any changes in the
thermodynamics properties of the universe is called a
nRT reversible process.
p=  can be reversed completely and there is no trace left to
V show that the system had undergone thermodynamic
change.
Work done in a volume change at constant pressure
 occur in infinitesimally small time
W =p ( V 2−V 1 )  all of the initial and final state of the system should be in
equilibrium with each other

Efficiency of a Heat Engine If during the reversible process the heat content of the system
remains constant, i.e. it is adiabatic process, then the process is also
Efficiency measures how well an isentropic process, i.e. the entropy of the system remains constant.
engine operates
A cyclic process cannot completely The phenomenon of undergoing reversible change is also called
convert energy transferred as heat into reversibility. In actual practice the reversible process never occurs,
work, nor can it transfer energy as heat thus it is an ideal or hypothetical process.
from a low-temperature body to a high-
temperature body without work being Irreversible Process
done in the process. However, a cyclic  Also known as Natural Processes
process can be made to approach these  Thermodynamic processes that occur in nature are all
ideal situations. A measure of how well
irreversible processes.
an engine operates is given by the
engine’s efficiency (eff). In general,  These are processes that proceed spontaneously in one
efficiency is a measure of the useful direction but not the other.
energy taken out of a process relative to  For instance, heat flow between two bodies occurs due to
the total energy that is put into the the temperature gradient between the two bodies; this is in
process. Efficiencies for different types fact the natural flow of heat. Similarly, water flows from
of high level to low level, current moves from high potential
engines are listed in the table. to low potential, etc.

35 | P a g
e
 The initial state of the system and surroundings cannot be 2. if we let the gas expand, pushing the piston upward, the volume
restored from the final state. of the gas will __________ so the state must shift to the
 The various states of the system on the path of change _______ toward larger volumes. Since the gas is expanding,
from initial state to final state are not in equilibrium with we can also say for sure that ____________ work (W) is being
each other. done on the gas.
 The entropy of the system increases decisively and it
cannot be reduced back to its initial value.
Activity 4: Direction: Solve the following problems. Show your
 The phenomenon of a system undergoing irreversible
solutions.
process is called as irreversibility.
1. A heat engine does 340 J of work during which its internal
energy decreases by 500 J. what is the net heat exchange for this
process?

Activities: 2. Assume that the gas inside the cylinder expands under a
constant pressure of 250 kPa, while its volume increases from 2.5 x
Activity 1: Direction: Answer the following questions based on 10 -3 m3 to 5.6 x 10 -3 m3. What work is done by this gas?
your understanding.
3. In an industrial chemical process, 500 J of heat is supplied
1. Identify and describe the different properties of ideal gas. Use to a system and 250 J of work is done by the system. What is the
a table for your answer. increase in the internal energy of the system?
Activity 2: Direction: Show your solution in solving the 4. In a thermodynamic process, the internal energy of the
unknown quantity. system increases by 450 j. how much work was done by the gas if
850 J of heat is absorbed?
1. What volume of hydrogen gas at atmospheric pressure is
required to fill a 6000 ml tank under a gauge pressure of 5. A system absorbs 200 J of heat as he internal energy
540 kPa? increase by 180 J. What work is done by the gas?

2. A frictionless cylinder is filled with 1.5 L of an ideal gas at


22oC. One end of the cylinder is fixed with a movable piston, Activity 5: Direction: Answer the following questions.
and the gas is allowed to expand at constant pressure until is
volume reaches 2.0 L. what is the new temperature of the gas? 1. A gas undergoes an adiabatic expansion. Does it perform
external work? If so, what is the source of energy?

3. An automobile is inflated to a gauge pressure of 209 kPa at a 2. A gas performs external work during an isothermal
time when the surrounding pressure is 1 atm and the expansion, what is the source of energy?
temperature is 23oC. After the car is driven, the temperature of
3. What happens to the internal energy of a gas undergoing
the air in the tire increases to 50 oC. Assuming the volume
a. Adiabatic compression,
changes only slightly, what will be the new gauge pressure in
b. Isothermal expansion,
the tire?
c. A throttling process?

4. An ideal gas occupies a volume of 5.0 m 3 at 250 kPa absolute Activity 6: Direction: complete the table to be able to
pressure, what will be the new pressure if the gas differentiate the four thermodynamic processes.
5. is slowly compressed to 2.0 m3 at constant temperature?

6. Five hundred cubic centimeters of an ideal gas ay 23oC


expands to a volume of 240 cm 3 at constant pressure, what is
the final temperature?

Activity 3: Direction: Analyze the diagram and complete the


paragraph that follows.

Activity 7: Direction: Solve the efficiency of heat engine. (5


points each problem. Show your solution)
1. An ideal engine, operating between two heat reservoirs at
500 K and 400 K, absorbs 1000 J of heat from the high temperature
reservoir during each cycle. What is the efficiency of the engine
and how much heat is rejected to the environment?
2. What is the efficiency of an engine that does 250 J of work
in each cycle while discarding 700 J to the environment?
3. A 40 percent efficient engine loses 450 J of heat during
each cycle. What work is done, and how much heat is absorbed in
each cycle?
4. A steam engine takes superheated steam from
1. If we press the piston downward, the volume of the gas will and rejects it directly into the air at 110oC. What is the ideal
__________ so the state must shift to the ______ toward efficiency?
smaller volumes. Since the gas is being compressed, we can
also say for sure that _______ work (W) is being done on the
gas.

36 | P a g
e
REFERENCES "Pascal’s principle | Definition, Example, & Facts | Britannica."
https://www.britannica.com/science/Pascals-principle.

"Principles of Flight: Bernoulli's Principle."


"Angular Speed Formula with Solved examples."
https://www.nasa.gov/sites/default/files/atoms/files/bernoulli_princi
https://byjus.com/angular-speed-formula/.
ple_k-4.pdf.
"PHYSICS 111 HOMEWORK SOLUTION #9."
"Bernoulli's principle - Wikipedia."
https://web.njit.edu/~gary/111/assets/HW9_SOL.pdf.
https://en.wikipedia.org/wiki/Bernoulli%27s_principle.
"In-Class Worksheet #14."
"What is the ideal gas law? (article) | Khan Academy."
http://clas.sa.ucsb.edu/staff/vince/physics%206a/Physics%206A
https://www.khanacademy.org/science/physics/thermodynamics/te
%20Ch9Worksheet%20solutions.pdf.
mp-kinetic-theory-ideal-gas-law/a/what-is-the-ideal-gas-law.
"Rotational inertia (article) | Khan Academy."
"11.9: The Ideal Gas Law: Pressure, Volume, Temperature ...." 01
https://www.khanacademy.org/science/physics/torque-angular-
Jul. 2019,
momentum/torque-tutorial/a/rotational-inertia.
https://chem.libretexts.org/Courses/can/intro/11%3A_Gases/11.09
"Determining the Net Force - Physics Classroom." %3A_The_Ideal_Gas_Law%3A_Pressure%2C_Volume
https://www.physicsclassroom.com/Class/newtlaws/U2l2d.cfm. %2C_Temperature%2C_and_Moles.

"Thermodynamic foundations of physical chemistry ...." 10 Dec.


2018, https://link.springer.com/article/10.1007/s10698-018-09324-
1.

"Newton's Third Law of Motion - Physics Classroom."


https://www.physicsclassroom.com/class/newtlaws/Lesson-
4/Newton-s-Third-Law.

"Chegg - Save up to 90% on Textbooks | Don't Pay Full Price ...."


https://www.chegg.com/.

"Simple Harmonic Motion - Cyberphysics - SHM."


https://www.cyberphysics.co.uk/topics/shm/shm.htm.

"Simple harmonic motion - Wikipedia."


https://en.wikipedia.org/wiki/Simple_harmonic_motion.

"Simple harmonic motion | Tree of Knowledge Wiki | Fandom."


https://tok.fandom.com/wiki/Simple_harmonic_motion.

"Simple Harmonic Motion (SHM) - Definition, Equations ...."


https://byjus.com/jee/simple-harmonic-motion-shm/

"Physics Flashcards | Quizlet."


https://quizlet.com/43171019/physics-flash-cards/.

"Sign Up - Course Hero." https://www.coursehero.com/register/.

"Coriolis force - Wikipedia."


https://en.wikipedia.org/wiki/Coriolis_force.

"Blog | LoganThrasherCollins.com | Page 2." 27 Nov. 2020,


https://logancollinsblog.com/blog/page/2/.

"(PDF) Walter Fox Smith Waves and oscillations A ...."


https://www.academia.edu/32704406/Walter_Fox_Smith_Waves_a
nd_oscillations_A_preluBookZZ_org.

"(PDF) David Morin Introduction to Classical Mechanics With ...."


https://www.academia.edu/43410742/David_Morin_Introduction_t
o_Classical_Mechanics_With_Problems_and_Solutions.

"Principle of Superposition - Definition, Formula ...."


https://byjus.com/physics/superposition-principle-and-continuous-
charge-distribution/.

"Principle of Superposition - San Jose State University."


https://www.sjsu.edu/faculty/beyersdorf/Archive/Phys158F06/10-
19%20Principle%20of%20Superposition.pdf

"Chapter 12 Properties of matter- Section 2 Guided Reading ...."


https://quizlet.com/123472115/chapter-12-properties-of-matter-
section-2-guided-reading-flash-cards/.

"Pascal’s Law: Applications & Examples – StudiousGuy."


https://studiousguy.com/pascals-law-applications-examples/.
37 | P a g
e

You might also like